Нок 25 20: Mathway | Популярные задачи

2

Наименьшее общее кратное 20 и 25

Калькулятор «Наименьшее общее кратное»

Какое наименьшее общее кратное (НОК) у чисел 20 и 25?

Ответ: НОК чисел 20 и 25 это 100

(сто)

Нахождение наименьшего общего кратного для чисел 20 и 25 используя НОД этих чисел

Первый способ нахождения НОК для чисел 20 и 25 — через нахождение наибольшего общего делителя (НОД) этих чисел. Формула:

НОК = (Число1 × Число2) ÷ НОД

НОД чисел 20 и 25 равняется 5, следовательно

НОК = (20 × 25) ÷ 5

НОК = 500 ÷ 5

НОК = 100

Нахождение наименьшего общего кратного для чисел 20 и 25 используя перечисление кратных

Второй способ нахождения НОК для чисел 20 и 25 заключается в перечислении всех кратных для обоих чисел и выбор первого совпадающего:

Кратные числа 20: 20, 40, 60, 80, 100, 120, 140

Кратные числа 25: 25, 50, 75, 100, 125, 150

Следовательно, НОК для 20 и 25 равняется 100

Нахождение наименьшего общего кратного для чисел 20 и 25 используя разложение чисел на простые множители

Еще один способ нахождения НОК чисел 20 and 25 — это нахождение всех простых множителей для обоих чисел и перемножение самых больших экспоненциальных форм

Похожие расчеты

Поделитесь текущим расчетом

Печать

https://calculat. io/ru/number/least-common-multiple-lcm-of/20—25

<a href=»https://calculat.io/ru/number/least-common-multiple-lcm-of/20—25″>Наименьшее общее кратное 20 и 25 — Calculatio</a>

О калькуляторе «Наименьшее общее кратное»

Данный калькулятор поможет найти Наименьшее общее кратное двух чисел. Например, он может помочь узнать какое наименьшее общее кратное (НОК) у чисел 20 и 25? Выберите первое число (например ’20’) и второе число (например ’25’). После чего нажмите кнопку ‘Посчитать’.

Наименьшее общее кратное (НОК) для двух чисел - это наименьшее натуральное число, которое делится на оба числа без остатка

Калькулятор «Наименьшее общее кратное»

Таблица Наименьших общих кратных

Число 1Число 2НОК
52525
625150
725175
825200
925225
102550
1125275
1225300
1325325
1425350
152575
1625400
1725425
1825450
1925475
2025100
2125525
2225550
2325575
2425600
252525
2625650
2725675
2825700
2925725
3025150
3125775
3225800
3325825
3425850

Цена акции

NOC | Котировка акций Northrop Grumman Corp.

(США: NYSE)
  • Тикер поиска

  • |

    НОК США: NYSE

    NOCUS

    Нерабочее время

    Последнее обновление: 26 мая 2023 г., 19:55. EDT Задержка котировки

    0,620,14%

    Объем в нерабочее время: 33,42К

    Закрыть Изменить Изменить %
    438,32 $ 6,68 1,55%

    Переключить параметры диаграммы

    116% по сравнению со средним

    Настроить MarketWatch

    Есть списки наблюдения? Войдите, чтобы увидеть их здесь, или зарегистрируйтесь, чтобы начать.

    Зарегистрироваться … или Войти

    Нет элементов в списке наблюдения

    В настоящее время в этом списке наблюдения нет элементов.

    Добавить тикеры

    Угу

    Что-то пошло не так при загрузке списка наблюдения.

    Перейти к списку наблюдения

    Нет последних тикеров

    Посетите страницу с котировками, и здесь будут отображаться недавно просмотренные тикеры.

    • Открыть 432,62 доллара США
    • Дневной диапазон 432,62 — 441,51
    • 52-недельный диапазон 429,10 — 556,27
    • Рыночная капитализация 66,56 млрд долларов
    • Акции в обращении 151,86 млн
    • Общественный поплавок 151,42 млн
    • Бета 0,62
    • Оборот на сотрудника 390,59 тыс. долларов
    • Коэффициент цена/прибыль 14.18
    • EPS $30,91
    • Выход 1,71%
    • Дивиденд 1,87 доллара США
    • Экс-дивидендная дата 26 мая 2023 г.
    • Короткие проценты 1,36 млн 15.05.23
    • % поплавка закорочено 0,90%
    • Средний объем 747,89 тыс.
    5 День
    • -1,13%
    1 месяц
    • -4,98%
    3 месяца
    • -6,92%
    с начала года
    • -19,66%
    1 год
    • -6,89%
    • Продам
    • Ниже
    • Удерживать
    • Более
    • Купить
    Количество оценок 22 Полные рейтинги

    Нет доступных заголовков

    Нет доступных заголовков

    Нет доступных заголовков

    Нет доступных заголовков

    Нет доступных заголовков

    Нет доступных заголовков

    Northrop Grumman Corp. занимается поставкой передовых авиационных систем. Он работает в следующих сегментах: системы аэронавтики, оборонные системы, системы миссий и космические системы. Сегмент Aeronautics Systems занимается проектированием, разработкой, производством, интеграцией, поддержкой и модернизацией передовых авиационных систем для ВВС США, ВМС США, других правительственных учреждений США и международных клиентов. Сегмент оборонных систем включает в себя интегрированные системы управления боем, системы вооружения и самолеты, а также поддержку и модернизацию систем миссии. Сегмент Mission Systems предлагает передовые решения для миссий и многофункциональные системы, в первую очередь для оборонного и разведывательного сообщества США, а также для международных клиентов. Сегмент космических систем предоставляет комплексные решения для миссий посредством проектирования, разработки, интеграции, производства и эксплуатации космических, противоракетных, пусковых и стратегических ракетных систем для национальной безопасности, гражданского правительства, коммерческих и международных клиентов. Компания была основана Джоном К. Нортропом, Томасом В. Джонсом и Кентом Крезой в 1939, со штаб-квартирой в Фолс-Черч, штат Вирджиния.

    Закрыть полосу текущих тикеров

    Доступ к премиум-инструментам

    NOC | Northrop Grumman Corp.

    Цены на акции и новости Реклама

    • НА ЗАКРЫТИЕ 16:00 EDT 26.05.23
    • $ 438,32 USD
    • 900 02 6,68 1,55%
    • Объем 865 024
    • Сравните с эталоном:
    • DJIA
    • S&P 500
    • GLOBAL DOW
    • NASDAQ
    • Промышленные товары
    • Сравните с
    • Открыто 432,62 900 07
    • Предварительное закрытие 431.64 (25.05.23)
    • 1 день
    • NOC 1,55%
    • DJIA 1,00%
    • S & P 500 1,30%
    • Промышленные товары 1,63%

    до

    Скачать электронную таблицу

    9 0046 23. 05.23 90 046 04.05.23 9004 6 459,59 9004 6 461,27 900 46 25.04.23 0 24.04.23 90 046 14.04. 23 9004 6 467,525 90 046 04.04.23 9 0046 461,72 9004 6 461,57 9 0046 449,98 90 046 449.11
    26.05.23 432,62 441,51 432,62 438,32 865 024
    25.05.23 438,33 439,95 429,10 433,51 910 608
    05 /24/23 443,03 446,35 441,38 441,44 664 637
    441,17 445,87 440,26 443,03 578 496
    22.05.23 442,09 445,18 438.435 443.07 508 740
    19.05.23 445.36 446.30 440,79 443,31 463 787
    18.05.23 446,07 446,92 437,75 9004 7 443,60 518 619
    17.05.23 439,00 448,40 437,58 447,34 758 396
    05 /16/23 439,99 440,3445 435,45 435,63 448 897
    15.05.23 438,75 440,08 436,00 439,37 491 027
    12.05.23 9004 7 437,09 441,8099 434,77 438,76 749 498
    05/11/23 438,90 440,31 433,18 435,85 699 017
    10. 05.23 443,10 445,00 438,70 443,03 559 606
    09.05.23 443,00 445,09 440,87 442.31 462 234
    08.05.23 448,33 450,95 441,87 442,24 418 159
    05 /05/23 443,46 448,45 441,93 446,81 474 749
    443,28 445,275 435,20 442,23 645 682
    03.05.23 900 47 447,53 452,28 443,57 443,71 520 165
    02.05.23 460,83 460,84 448,58 448,81 770 369
    01.05.23 461,95 466,0299 462,72 630 312
    28.04.23 460,37 461,57 455,525 708 800
    27. 04.23 452,48 459,675 443,00 457,68 1,37 М
    04/ 26/23 462,69 464,3281 449,07 449,09 1,41 М
    471,16 472,6305 466,50 470,92 593 594
    21.04.23 475,72 475,72 468,83 471,99 1,52 М
    20.04.23 478,40 478,40 47 2.07 474,18 511 686
    19.04.23 480,46 482,7909 476,20 90 047 477.07 482 811
    18.04.23 475,74 483,40 475,74 479,47 756 224
    04 /17/23 472,99 477,68 471,38 474,79 598 425
    470,55 473,09 468,765 472,57 501 938
    13.04.23 900 47 472,00 476,065 469,67 474.16 540 005
    12.04.23 467.43 474.7299 466. 03 472,77 574 938
    11.04.23 476,12 476,12 468,27 657 095
    10.04.23 470,98 478,17 470,98 476,54 594 820
    06.04.23 474,68 476,26 468,29 470,39 575 862
    04 /05/23 467,22 473,60 467,22 470,91 707 031
    468,73 471,94 465,00 468,10 495 274
    03.04.23 9004 7 463,30 471,55 462,65 469. 19 577 564
    31.03.23 462,98 463,32 459,45 640 613
    30.03.23 460,46 462,92 458,50 9 0047 460,98 541 518
    29.03.23 459,38 462,58 457,495 584 223
    28.03.23 458.31 461.50 457.59 459.00 476 830
    27.03.23 03 /24/23 445,08 456,20 443,00 455,25 1,01 М
    23.03.23 4 44,01 446,06 440,71 444,44 1,12 M
    22.03.23 450,22 452,40 443,085 4 43,60 726 223
    21.03.23 453,79 454,66 447,07 680 128
    20. 03.23 445,27 452,31 445,005 449. 26 662 434
    17.03.23 449.81 450,16 441,305 443,60 1,60 М
    16.03.23 455.97 448.00 450.15 1,08 М
    15.03.23 9004 7 448,97 456,38 439,87 448,90 1,51 М
    14.03.23 457,90 459,98 449,23 453,66 1,78 М
    13.03.23 454,69 462,87 453,715 457,74 1,24 М
    10.03.23 458,53 463,19 457,19 459 .78 772 293
    09.03.23 469,63 469,63 458,20 459,0 4 834 940
    08.03.23 472,00 475,63 460,92 465,25 1,09 М
    07.

    Размещение вероятность: Как различить сочетание и размещение

    Как различить сочетание и размещение

    Статьи › Школа › В ящике находится 15 деталей сколькими способами можно взять 4 детали

    Размещения — это упорядоченные наборы. Комбинации, при составлении которых важно знать только то, какие элементы выбраны, но их порядок не имеет значения, называются сочетаниями. В сочетаниях все элементы равноправны. Например, два дежурных, два куска хлеба.

    • Размещения — это наборы, в которых важен порядок элементов.
    • Сочетания — это наборы, в которых порядок элементов значения не имеет.
    • Перестановки — это наборы, в которых используются все элементы множества.
    • Число размещений из n элементов по k — это количество способов упорядоченного выбора k элементов из n.
    • Число сочетаний из n элементов по k — это количество способов неупорядоченного выбора k элементов из n.
    • Формула для вычисления числа размещений: A n k = n! / (n-k)!
    • Формула для вычисления числа сочетаний: C n k = n! / (k! * (n-k)!)
    • Можно помнить, что размещения — это как выбор команды из участников, где важна их позиция в команде, а сочетания — это как выбор команды из участников, где позиция участников значения не имеет.
    1. Что называют размещением
    2. Как считать размещение
    3. Как понять когда порядок важен
    4. Что такое число размещений
    5. Как не путать размещение и сочетание
    6. Как обозначается размещение
    7. Как определить число комбинаций
    8. Какие соединения называют размещением
    9. Что называется сочетаниями
    10. Что называется перестановки размещения сочетания
    11. Что такое размещение в теории вероятности
    12. Чем упорядоченная выборка отличается от неупорядоченной
    13. Что такое сочетание в теории вероятности
    14. Что такое размещение в информатике
    15. Что такое сочетание с повторениями
    16. Какие выборки называются размещениями
    17. Как понять Комбинаторику
    18. Что такое A из n по k
    19. Как найти число размещений из n по m
    20. Сколько комбинаций из 3 цифр с повторением
    21. Что такое n и m в комбинаторике
    22. Как считать перестановки
    23. Сколькими способами можно выбрать 3 из 10
    24. Что значит перестановка в математике

    Что называют размещением

    Ряд, заполненный объектами данного множества, называется размещением, т. е. мы разместили объекты на данных местах.

    Как считать размещение

    Формула числа размещений

    Akn=Ckn⋅k!

    Как понять когда порядок важен

    Порядок элементов важен, т. к., если поменять местами людей, обязанности их изменятся.

    Что такое число размещений

    Числом размещений из по (обозначается) называется количество способов расположить некоторые из различных объектов на пронумерованных местах, при условии что каждое место занято в точности одним объектом.

    Как не путать размещение и сочетание

    Теория:

    • Размещения — это упорядоченные наборы.
    • Комбинации, при составлении которых важно знать только то, какие элементы выбраны, но их порядок не имеет значения, называются сочетаниями. В сочетаниях все элементы равноправны. Например, два дежурных, два куска хлеба.
    • Сочетания не являются упорядоченными наборами.

    Как обозначается размещение

    Размещения вычисляются по формуле A n m = n! (n − m)!.

    Как определить число комбинаций

    Количество сочетаний обозначается как C n m (читается: сочетания из \(n\) по \(m\)). Сочетания вычисляются по формуле C n m = n! M! (n − m)!.

    Какие соединения называют размещением

    Размещениями из n элементов по k называются соединения, которые можно образовать из n элементов, собирая в каждое соединение по k элементов, при этом соединения могут отличаться друг от друга как самими элементами, так и порядком их расположения.

    Что называется сочетаниями

    Каждое множество, содержащее m элементов из числа n заданных, называется сочетанием из n элементов по m. Подчеркнем, что сочетание определено как множество некоторых элементов без рассмотрения порядка, в котором они расположены. Число всех сочетаний из n элементов по m обозначается.

    Что называется перестановки размещения сочетания

    Классическими понятиями комбинаторики являются перестановки, размещения и сочетания. Перестановкой называется какой-либо способ упорядочения данного множества.

    Что такое размещение в теории вероятности

    Размещениями из n элементов по m (мест) называются такие выборки, которые имея по m элементов, выбранных из числа данных n элементов, отличаются одна от другой либо составом элементов, либо порядком их расположения. m = n·(n − 1)·(n − 2)·

    Чем упорядоченная выборка отличается от неупорядоченной

    Выборка называется упорядоченной, если порядок следования элементов в ней задан. Две упорядоченные выборки, отличающиеся лишь порядком следования элементов, считаются различными. Если порядок следования элементов не является существенным, то выборка называется неупорядоченной.

    Что такое сочетание в теории вероятности

    Определение. Неупорядоченные подмножества (комбинации), т. е. такие, порядок следования элементов в которых не играет роли и которые отличаются друг от друга хотя бы одним элементом, называются сочетаниями.

    Размещения — наборы последовательностей определенной длины, состоящие из определенных символов, которые могут встречаться в последовательности сколько угодно раз. Количество размещений N зависит от длины последовательности k и количества символов n как N=nk.

    Что такое сочетание с повторениями

    Определение: Сочетания с повторениями (англ. combinations with repetitions) — те же сочетания, только теперь даны типов элементов, из которых нужно выбрать элементов, причем элементов каждого типа неограниченное количество, и элементы одного типа должны стоять подряд друг за другом.

    Какие выборки называются размещениями

    Упорядоченные выборки объемом M из N элементов (M < N), где все элементы различны, называются размещениями. Число размещений из N элементов по M обозначается. Рассуждая иначе: первый элемент выбираем N способами, второй — (N — 1) способами и т. д., M–й элемент выбираем (N — M + 1) способом.

    Как понять Комбинаторику

    В узком смысле комбинаторика — это подсчёт различных комбинаций, которые можно составить из некоторого множества дискретных объектов. Под объектами понимаются какие-либо обособленные предметы или живые существа — люди, звери, грибы, растения, насекомые и т.

    Что такое A из n по k

    Число размещений Ank — расположение «предметов» на некоторых «местах» при условии, что каждое место занято в точности одним предметом и все предметы различны. Более формально, размещением (из n по k) называется упорядоченный набор из k различных элементов некоторого n-элементного множества.

    Как найти число размещений из n по m

    Формула для числа размещений. Размещениями из n элементов по m (мест) называются такие выборки, которые имея по m элементов, выбранных из числа данных n элементов, отличаются одна от другой либо составом элементов, либо порядком их расположения. Anm = n·(n − 1)·(n − 2)· ·(n − m + 1) = n!/(n − m)!

    Сколько комбинаций из 3 цифр с повторением

    3 = 60 способов расстановки цифр, т. е. искомое количество трехзначных чисел есть 60. (Вот некоторые из этих чисел: 243, 541, 514, 132, )

    Что такое n и m в комбинаторике

    Число размещений с повторением даётся формулой nm, число сочетаний с повторением — формулой Cmn+m-1.

    Как считать перестановки

    Количество перестановок обозначается как P n, где \(n\) — количество элементов множества. Перестановки вычисляются по формуле P n = n!

    Сколькими способами можно выбрать 3 из 10

    2) 12 + 3 = 15, Еще из 12 исключим один, прибавим 11 и 3 и получим 14 способов.

    Что значит перестановка в математике

    Перестановкой называются наборы, состоящие из одного и того же числа элементов, отличающихся только порядком следования элементов.

    Трафареты Яндекса — что это, как выбрать

    Мы используем cookie-файлы

    Поделиться термином «»

    Избранное

    Трафарет – это технология Яндекса, которая определяет внешний вид рекламной выдачи.

    До появления трафаретов объявления выглядели более единообразно, а рекламная выдача стандартно содержала 3-4 рекламных объявления в верхней части, 4 — в нижней. С появлением трафаретов объявления стали более гибкими: теперь они отображаются в той или иной части экрана, состоят из разных элементов и выглядят по-разному.

    Как выбрать трафарет

    Задать в Директе трафарет для объявления нельзя, точно так же нельзя отключить тот или иной трафарет. Система сама определяет, как будет выглядеть рекламная выдача в каждом конкретном случае. На выбор трафарета влияет как тематика запроса, так и поведение каждого отдельного юзера.

    Можно добавить в объявление как можно больше дополнительных элементов (уточнения, быстрые ссылки с описаниями, телефон и адрес, изображение и видео). Тогда у системы будет больше вариантов, как показать объявление.

    Также можно увеличить вероятность показа в эксклюзивном размещении или в саджесте — для этого нужно установить повышающие корректировки ставок.

    Какие бывают трафареты

    Яндекс регулярно тестирует разные форматы выдачи и объявлений.

    Обычные

    Объявления показываются со всеми добавленными расширениями: быстрыми ссылками, контактной информацией, отображаемой ссылкой. Если объявление показывается на первой позиции, в нем также отображаются уточнения.

    Минималистичные

    Объявления показываются без расширений. Это удешевляет стоимость размещения.

    Эксклюзивное размещение

    Объявление показывается над результатами поиска и занимает единственное рекламное место.

    Система рассматривает этот формат только для тех объявлений, в которых::

    Реклама в саджесте

    Объявление показывается под списком подсказок в тот момент, когда пользователь вводит запрос в поисковой строке. После показа в подсказке оно может отображаться и в результатах поиска.

    Дублирование объявления из спецразмещения в нижней части выдачи

    Объявление показывается в двух местах в выдаче: на первом месте в блоке «Премиум-показы» и на первой позиции в блоке «Остальные показы».

    При этом в статистику попадает только один показ и оплата будет списываться за один клик. Этот трафарет создан, потому что пользователи, просмотрев всю выдачу, нередко возвращаются наверх. Повторение первого объявления избавляет их от необходимости скроллить обратно и повышает вероятность целевого клика.

    Как посмотреть статистику по трафаретам

    Посмотреть статистику по показам в премиальном одиночном трафарете или в саджесте можно по срезу «Вид размещения» в Мастере Отчетов.

    Вам может быть интересно

    Ко всем статьям

    Согласие на обработку данных

    Биномиальное распределение

    — Вероятность бронирования отелей

    спросил

    Изменено 2 года, 7 месяцев назад

    Просмотрено 546 раз

    $\begingroup$

    В гостинице 100 номеров, и с гостей взимается предоплата. Количество бронирований на завтрашний вечер обозначается как $n$. {n-k}$. У меня возникли проблемы с попыткой создать функцию прибыли. Моя первая попытка была $$v(X) = \begin{case} 50n-60(X-100) & X > 100\\ 50n & X \leq 100 \end{cases}$$ Однако, чтобы максимизировать выигрыш, кажется, что отель должен правильно понимать, кто именно появится, что является случайным. Эта функция также, по-видимому, не учитывает вероятность появления гостей, что должно учитывать выбор, который сделает отель. Какой тип функции прибыли будет учитывать вероятность прихода гостей? 9n p(X=x)\cdot v(x)$$

    Мы можем написать сценарий R для оптимизации этой функции:

     # функция для определения ожидаемой прибыли, если они забронируют n номеров
    ожидаемая_прибыль <- функция (n) {
      Х = последовательность (0, п)
      вероятность = dbinom (X, размер = n, вероятность = 19/20)
      прибыль = ifelse(X>100, 50*n-60*(X-100), 50*n)
      возврат (сумма (вероятность * прибыль))
    }
    # построение этой функции
    сюжет(
      последовательность (1400),
      unlist(Map(ожидаемая_прибыль, seq(1400))),
      тип = "л"
      )
    # найти максимум
    which. max(unlist(Map(expected_profit, seq(1400))))
     

    Это дает результат $n=108$. То есть они оптимизируют свою среднюю прибыль, бронируя 108 номеров.

    $\endgroup$

    2

    $\begingroup$

    Очевидно, что оптимальное количество бронирований, приносящих максимальную прибыль, должно быть больше, чем количество доступных номеров (т.е. 100). Таким образом, нам нужно только найти оптимальное число бронирования с $n > 100$. 9{н-к}$.

    Это дает оптимальный номер резервирования 108 для $p=0,95$, такой же, как указано выше для PedroSebe .

    $\endgroup$

    Зарегистрируйтесь или войдите в систему

    Зарегистрируйтесь с помощью Google

    Зарегистрироваться через Facebook

    Зарегистрируйтесь, используя электронную почту и пароль

    Опубликовать как гость

    Электронная почта

    Требуется, но никогда не отображается

    Опубликовать как гость

    Электронная почта

    Требуется, но не отображается

    Нажимая «Опубликовать свой ответ», вы соглашаетесь с нашими условиями обслуживания, политикой конфиденциальности и политикой использования файлов cookie

    .

    Размещение – PDE и вероятностные методы взаимодействия

    Антиб: город, полный характера и очарования…

    Антиб привлекает посетителей со всего мира. Охраняемый курорт на Ривьере, Антиб благословлен прекрасным расположением между морем и горами. Этот очаровательный город привлекает посетителей широким спектром мероприятий. Идеально расположенный в самом сердце Лазурного берега, Антиб является настоящей жемчужиной Ривьеры. Старый город с крепостными стенами, знаменитым музеем Пикассо и живописными узкими улочками выходит на море и предлагает захватывающий вид на Средиземное море.


    Все участники будут размещены в отеле ROYAL ANTIBES (отель 4*)

    Этот отель имеет очень удобное расположение: 25 минут езды от международного аэропорта Ниццы или 20 минут от Канн и 15 минут от Софии. Технополис Антиполис. Его лаконичный современный фасад выходит на знаменитую бухту Иллетт, одну из самых красивых на Лазурном берегу, с песчаным пляжем и захватывающим дух видом на Кап д’Антиб.

    Все номера и апартаменты оснащены совершенно новой мебелью, кондиционерами, плазменными телевизорами и бесплатным Wi-Fi и обеспечивают идеальное сочетание комфорта и современного дизайна.

    Будет организован трансфер от отеля Royal до Инрии (и обратно) в четверг 30 и в пятницу 31.

    Адрес :
    ROYAL ANTIBES
    16 Boulevard Maréchal Leclerc
    06600 Antibes (Франция)
    +33 4 83 61 91 91
    [email protected]


               


    Как добраться до отеля Royal?
    • Из аэропорта Ницца Лазурный Берег
    • От железнодорожного вокзала Антиб
    • На машине

    Международный аэропорт Ницца Лазурный Берег находится в 15 км от Антиба. Переезды в город занимают в среднем 25 минут на машине или такси, час на общественном транспорте. http://en.nice.aeroport.fr/

    На автобусе: 1,50 €
    Автобус № 200 (в направлении города Канны).
    — посадка на остановке NICE — Aéroport / Promenade (терминал 1)
    — Сойти на остановке ANTIBES – Pôle d’échange Antibes.
    — Затем сядьте на автобус 14 и сойдите на остановке Мезьер в 80 метрах от отеля.

    На машине:
    Координаты GPS: широта 43,574997/долгота 7,124258
    Парковка на месте

    На такси: ориентировочная стоимость 55 евро
    — Трансфер (на английском языке) – специальные тарифы для конференции PDE
    +33 (0) 6 09 50 92 53
    — Central Taxi Riviera Nice +33 (0) 4 93 13 78 78
    -Мототранспорт: +33 (0) 6 58 7981 31

    Вокзал скоростных поездов Antibes, откуда можно добраться до всех крупных европейских городов, находится в 15 минутах ходьбы от отеля.

    Во дворе автобусная остановка; просто сядьте на автобус 14 и сойдите на остановке Мезьер в 80 метрах от отеля.

    Информация SNCF (железнодорожный вокзал)
    www.

    2 см перевести в м: Сантиметры в метры | Онлайн калькулятор

    Перевод единиц измерения

    В этой статье поговорим о физических величинах, с которыми приходится иметь дело в сопромате и их единицах измерения. А главное, как эти единицы измерения переводить из одной в другую. Так как я знаю, что у многих студентов (особенно у чайников), с этим возникают проблемы, а как следствие ошибки в расчётах.

    Если ты пока не очень хорошо ориентируешься в размерностях, можешь использовать эту страничку, как шпаргалку. Я постарался собрать здесь все, что тебе может понадобиться при выполнении расчетов.

    Перевод единиц длины (расстояния)

    Начнём, пожалуй, с самого распространённого, с чем приходится работать при решении задач по сопромату – с длиной.

    В расчётах ты можешь встретить: метры (м), сантиметры (см) и миллиметры (мм).

    Скорее всего, ты уже знаешь, как перевести одно в другое, но для полноты картины приведу эту информацию.

    Перевод метров (м)

    метры (м) в сантиметры (см)
    1 м = 100 см

    или

    1 м = 1
    · 102 см
    метры (м) в миллиметры (мм)
    1 м = 1000 мм

    или

    1 м = 1
    · 103 мм

    Перевод сантиметров (см)

    сантиметры (см) в метры (м)
    1 см = 0.01 м

    или

    1 см = 1
    · 10-2 м
    сантиметры (см) в миллиметры (мм)
    1 см = 10 мм

    или

    1 см = 1
    · 10 мм

    Перевод миллиметров (мм)

    миллиметры (мм) в сантиметры (см)
    1 мм = 0.1 см

    или

    1 мм = 1
    · 10-1
    миллиметры (мм) в метры (м)
    1 мм = 0.001 м

    или

    1 мм = 1
    · 10-3 м

    Примеры перевода единиц длины

    К примеру, ты получаешь в ответе метры:

    l = 0.028 м

    И тебе нужно перевести это в миллиметры, твои действия следующие:

    l = 0. 028 м = 0.028 · 103 мм = 28 мм

    Или, наоборот, нужно перевести миллиметры в метры, тогда:

    l = 28 мм = 28 · 10-3 м = 0.028 м

    Перевод единиц площади

    В расчётах тебе часто придётся рассчитывать площадь какого-либо поперечного сечения и конвертировать это значение в различные единицы. Это может быть площадь простой фигуры, как здесь. А также площадь составного сечения, состоящего из простых, как здесь.

    Перевод кв. метров (м

    2)
    кв. метры (м
    2) в кв. сантиметры (см2)
    1 м
    2 = 10000 см2

    или

    1 м
    2 = 1 · 104 см2
    кв. метры (м
    2) в кв. миллиметры (мм2)
    1 м
    2 = 1000000 мм2

    или

    1 м
    2 = 1 · 106 мм2

    Перевод кв. сантиметров (см

    2)
    кв.
    сантиметры (см2) в кв. метры (м2)
    1 см
    2 = 0.0001 м2

    или

    1 см
    2 = 1 · 10-4 м2
    кв. сантиметры (см
    2) в кв. миллиметры (мм2)
    1 см
    2 = 100 мм2

    или

    1 см
    2 = 1 · 102 мм2

    Перевод кв. миллиметров (мм

    2)
    кв. миллиметры (мм
    2) в кв. сантиметры (см2)
    1 мм
    2 = 0.01 см2

    или

    1 мм
    2 = 1 · 10-22
    кв. миллиметры (мм
    2) в кв. метры (м2)
    1 мм
    2 = 0.000001 м2

    или

    1 мм
    2 = 1 · 10-6 м2

    Примеры перевода единиц площади

    Давай переведем квадратные сантиметры в квадратные миллиметры:

    A = 2. 2 см2 = 2.2 · 102 мм2 = 220 мм2

    И, наоборот, мм2 в см2:

    A = 220 мм2 = 220 · 10-22 = 2.2 cм2

    Перевод единиц момента сопротивления

    При решении задач по сопромату, чаще всего, с кубическими единицами (м3, см3, мм3) тебе придется сталкиваться при расчете моментов сопротивления. Иногда, при расчете статических моментов инерции и, конечно, объемов.

    Перевод куб. метров (м

    3)
    куб. метры (м
    3) в куб. сантиметры (см3)
    1 м
    3 = 1000000 см3

    или

    1 м
    3 = 1 · 106 см3
    куб. метры (м
    3) в куб. миллиметры (мм3)
    1 м
    3 = 1000000000 мм3

    или

    1 м
    3 = 1 · 109 мм3

    Перевод куб.

    сантиметров (см3)
    куб. сантиметры (см
    3) в куб. метры (м3)
    1 см
    3 = 0.000001 м3

    или

    1 см
    3 = 1 · 10-6 м3
    куб. сантиметры (см
    3) в куб. миллиметры (мм3)
    1 см
    3 = 1000 мм3

    или

    1 см
    3 = 1 · 103 мм3

    Перевод куб. миллиметров (мм

    3)
    куб. миллиметры (мм
    3) в куб. сантиметры (см3)
    1 мм
    3 = 0.001 см3

    или

    1 мм
    3 = 1 · 10-3 см3
    куб. миллиметры (мм
    3) в куб. метры (м3)
    1 мм
    3 = 0.000000001 м3

    или

    1 мм
    3 = 1 · 10-9 м3

    Пример перевода единиц момента сопротивления

    Для примера переведем кубические миллиметры в кубические сантиметры:

    W = 328 мм3 = 328 · 10-3 см3 = 0. 328 см3

    И, обратно:

    W = 0.328 см3 = 0.328 · 103 мм3 = 328 мм3

    Перевод единиц момента инерции

    Вот мы и до моментов инерции добрались. Хочу отметить, что речь будет идти о моментах инерции плоских сечений (фигур), которые приходится часто считать в курсе «сопротивление материалов».

    Перевод метров в 4 степени (м

    4)
    м
    4 в см4
    1 м
    4 = 100000000 см4

    или

    1 м
    4 = 1 · 108 см4
    м
    4 в мм4
    1 м
    4 = 1000000000000 мм4

    или

    1 м
    4 = 1 · 1012 мм4

    Перевод сантиметров в 4 степени (см

    4)
    см
    4 в м4
    1 см
    4 = 0.00000001 м4

    или

    1 см
    4 = 1 · 10-8 м4
    см
    4 в мм4
    1 см
    4 = 10000 мм4

    или

    1 см
    4 = 1 · 104 мм4

    Перевод миллиметров в 4 степени (мм

    4)
    мм
    4 в см4
    1 мм
    4 = 0. 0001 см4

    или

    1 мм
    4 = 1 · 10-4 см4
    мм
    4 в м4
    1 мм
    4 = 0.000000000001 м4

    или

    1 мм
    4 = 1 · 10-12 м4

    Пример перевода единиц момента инерции

    Здесь, давай переведем м4 в см4:

    I = 0.0412 · 10-3 м4 = 0.0412 · 10-3· 108 см4 = 4120 см4

    И в обратную сторону:

    I = 4120 см4 = 4120 · 10-8 м4 = 0.0412· 10-3 м4

    Единицы измерения силы

    Сила измеряется в Ньютонах (Н) и обычно указывается сразу в килоНьютонах (кН).

    F = 20 кН

    Приставка кило – это множитель 103, а сила в Ньютонах будет равна:

    F = 20 кН = 20 · 103 Н

    Также тебе часто придется иметь дело с распределенными нагрузками:

    q = 10 кН/м

    Которую тоже можно перевести как:

    q = 10 кН/м = 10 · 103 Н/м

    Перевод единиц напряжения

    И, наконец, тебе понадобится информация по единицам напряжения. Напряжение измеряется в Паскалях (Па), чаще всего оно указывается сразу в МегаПаскалях (МПа).

    Например:

    σ = 160 МПа

    Напомню, приставка Мега (М) означает множитель 106. То есть чтобы перевести это значение просто в Па, нужно умножить на 106:

    σ = 160 МПа = 160 · 106 Па

    Так как, 1 Па = 1 Н/м2, то:

    σ = 160 МПа = 160 · 106 Па = 160 · 106 Н/м2

    Также это значение можно перевести в такие единицы:

    σ = 160 МПа = 160 · 106 Н/м2 = 160 · 102 Н/см2 = 160 Н/мм2

    Примеры перевода единиц измерения

    Теперь предлагаю применить все, о чем я писал в этой статье на практике.

    Скажем, у нас есть формула:

    Эта формула позволяет вычислить изменение длины стержня, работающего на растяжение-сжатие.

    Также известны величины:

    N = 20 кН; l = 20 мм; A = 2 см2; E = 2·105 МПа.

    Чтобы получить правильный ответ при подстановке численных значений, нужно преобразовать все подставляемые значения к одному виду: например, к Ньютонам и метрам.

    Давай сделаем это для каждой величины отдельно:

    N = 20 кН = 20 · 103 Н

    l = 20 мм = 20 · 10-3 м = 0.02 м

    E = 2·105 МПа = 2·105·106 Па = 2·1011 Па = 2·1011 Н/м2.

    A = 2 см2 = 2 · 10-4 м2

    Подставим преобразованные значения в формулу:

    Или:

    Проанализируем размерности подставленных значений:

    Видим, что м2 и Н, в левой части сократятся и останутся только метры. Таким образом, можно проверять себя при подстановке значений в формулу.

    Другие уроки для чайников, размещенные на ssopromat.ru, можно найти, перейдя по указанной ссылке.

    Перевести Сантиметры в Метры

    Перевести Сантиметры в Метры

      

    Выберите тип перевода:
    сантиметры => метрыметры => сантиметры

    Округление результата:
    1 цифра после запятой2 цифры после запятой3 цифры после запятой4 цифры после запятой5 цифр после запятой6 цифр после запятой7 цифр после запятой8 цифр после запятой9 цифр после запятой10 цифр после запятой



    Перевести Метры в Сантиметры (m в cm) ►

    Таблица преобразования

    сантиметры в метры
    cmm
    10 cm 0. 1 m
    20 cm 0.2 m
    30 cm 0.3 m
    40 cm 0.4 m
    50 cm 0.5 m
    60 cm 0.6 m
    70 cm 0.7 m
    80 cm 0.8 m
    90 cm 0.9 m
    100 cm 1 m
    110 cm 1. 1 m
    120 cm 1.2 m
    130 cm 1.3 m
    140 cm 1.4 m
    150 cm 1.5 m
    160 cm 1.6 m
    170 cm 1.7 m
    180 cm 1.8 m
    190 cm 1.9 m
    200 cm 2 m

    Как переводить

    1 сантиметр (cm) = 0. 01 m (метр). Единица измерения сантиметр (cm) — это мера длины, которая используется в метрической системе. Единица измерения метр (m) — это мера длины, которая используется в метрической системе. Сантиметры также могут обозначатся как centimetres (вариант написания в Великобритании).

    English     Español     Français


    Конвертация единиц измерения:

    сантиметры в дециметры
    сантиметры в футы
    сантиметры в дюймы
    сантиметры в метры
    сантиметры в миллиметры
    сантиметры в ярды
    метры в сантиметры
    метры в дециметры
    метры в футы
    метры в дюймы
    метры в километры
    метры в мили
    метры в миллиметры
    метры в ярды
    сантиметры в дюймы
    футы в дюймы
    футы в километры
    футы в метры
    футы в ярды
    дюймы в сантиметры
    дюймы в футы
    дюймы в метры
    дюймы в миллиметры
    километры в мили
    метры в футы
    метры в дюймы
    метры в ярды
    мили в километры
    миллиметры в дюймы
    ярды в футы
    ярды в дюймы
    ярды в метры


    Сообщить об ошибке на этой странице

    Сколько метров в 2 см?

    Свяжитесь с нами!

    Пожалуйста, свяжитесь с нами, если вы:

    1. Есть предложения
    2. Есть вопросы
    3. Нашли ошибку/ошибку
    4. Что-нибудь еще. ..

    Чтобы связаться с нами, нажмите ЗДЕСЬ.

    2 сантиметра равны 0,02 метра, потому что 2 умножить на 0,01 (коэффициент преобразования) = 0,02

    Универсальный конвертер

    Найдите другие преобразования здесь:

    Определение сантиметра

    A сантиметр (см) — десятичная дробь метра, единица длины в Международной системе единиц (СИ), примерно равная 0,39 дюйма. Сантиметр (см) — единица длины, которая ранее была базовой единицей длины в системе единиц сантиметр-грамм-секунда (СГС). Хотя технические специалисты предпочитают префиксы SI для коэффициентов 10 3 , сантиметр остается практической единицей длины для повседневных измерений. Ширина ногтя среднего взрослого человека примерно равна одному сантиметру, что делает его удобным для измерения небольших предметов или расстояний. Узнайте больше о сантиметре и его использовании в повседневной жизни.

    Вот несколько примеров вещей размером около одного сантиметра (порядок величины):

    Ширина ногтя взрослого человека
    Диаметр стандартного стержня карандаша
    Толщина кредитной карты
    Высота маленького кубика Lego
    Диаметр батарейки АА
    Длина муравья
    Ширина маленькая кнопка
    Толщина защитной пленки для экрана смартфона
    Ширина рисового зерна
    Высота маленькой скрепки.

    Определение метра

    Метр – это длина пути, пройденного светом в вакууме за промежуток времени 1/299 792 458 секунды. Один метр примерно на 3 3/8 дюйма длиннее ярда, то есть примерно 39 3/8 дюйма.

    Вот некоторые распространенные способы преобразования метров в другие единицы измерения длины:

    1 метр = 100 сантиметров
    1 метр = 3,28084 фута
    1 метр = 1,09361 ярда
    1 метр = 0,000621371 мили
    1 метр = 39,3701 дюйма

    И наоборот, чтобы преобразовать эти другие единицы длины в метры, вы должны использовать соответствующее преобразование коэффициент, либо путем умножения, либо деления исходной величины на коэффициент.

    Таким образом, метр является единицей длины в системе СИ и обычно используется для измерения расстояния и длины в различных контекстах. Его основа в единицах 10 позволяет легко преобразовать его в другие единицы длины.

    Вот несколько примеров вещей размером около одного метра (порядка величины):

    Типичный размах человеческих рук
    Метровая палка или линейка
    Размер велосипедной рамы
    Большая пицца
    Рыба длиной три фута (1 метр)
    Стандартная кухонная столешница
    Собака среднего размера
    Высота баскетбольного кольца
    Длина типичного бильярдного кия
    Стандартная трость для ходьбы
    Небольшая лестница или табурет-ступенька
    Микроволны с частотой 300 ГГц имеют длину волны 1 мм

    Как преобразовать 2 сантиметра в метры

    Чтобы рассчитать значение в метрах, вам просто нужно использовать следующую формулу :

    Значение в метрах = значение в сантиметрах × 1 / 100

    Другими словами, вам нужно умножить значение емкости в сантиметрах на 1 / 100 для получения эквивалента в метрах.

    Например, чтобы преобразовать два см в метры, вы можете подставить значение 2 в приведенную выше формулу, чтобы получить

    метра = 2 × 1 / 100 = 0,02

    Следовательно, емкость конденсатора равна 0,02 метр. Обратите внимание, что полученное значение, возможно, придется округлить до практического или стандартного значения, в зависимости от приложения.

    Используя этот конвертер, вы можете получить ответы на такие вопросы, как:

    • Сколько 2 сантиметра в метрах;
    • Как перевести сантиметры в метры и
    • Какая формула для перевода сантиметров в метры, среди прочего.

    Сантиметры в Метры.

    1,5 сантиметра равно 0,015 метра
    1,6 сантиметра равно 0,016 метра
    1,7 сантиметра равно 9002 8 0,017 метра
    1,8 сантиметра равно 0,018 метра
    1,9 сантиметра равно 0,019 метра
    2 сантиметра равно 0,02 метра
    2,1 сантиметра равно 0,021 метра
    2,2 сантиметра равно 0,022 метра
    2,3 сантиметры равно 0,023 метра
    2,4 сантиметра равно 0,024 метра
    2,5 сантиметра равно 0,025 метра
    2,6 сантиметра равно 0,026 метра

    Примечание. Значения округлены до 4 значащих цифр. Дроби округляются до ближайшей восьмой дроби.

    Преобразование образцов

    Отказ от ответственности

    Несмотря на усилия по предоставлению точной информации на этом веб-сайте, ее точность не гарантируется. Поэтому контент не должен использоваться для принятия решений, касающихся здоровья, финансов или имущества.

    О нас | Свяжитесь с нами | Конфиденциальность
    Copyright © 2016 — 2023 HowMany.wiki

    Преобразовать 2 сантиметра в метры

    Какой длины 2 сантиметра? Как далеко 2 сантиметра в метрах? Преобразование 2 см в м.

    От АнгстремыСантиметрыГаземыНогиФурлонгиДюймыКилометрыМетрМикроныМилиМиллиметрыНанометрыМорские милиПикометрыЯрды

    До Ангстремы Сантиметры Морские сажени Ноги Фурлонги Дюймы Километры Метры Микроны Мили Миллиметры Нанометры Морские мили Пикометры Ярды 2 сантиметра =

    0,02 метра

    (точный результат)

    Показать результат как NumberFraction (точное значение)

    Сантиметр или сантиметр — это единица длины, равная одной сотой метра.

    Онлайн калькулятор решения уравнений с модулем: Калькулятор онлайн — Решение уравнений и неравенств с модулями

    Уравнения с модулями 6 класс :: mymalama

    17.12.2021 02:59

     

     

     

     

     

     

     

     

     

     

     

     

     

     

     

     

     

     

     

     

    Автор: Голованов А. С. Упорядочивание по возрастанию убыванию. Уравнения, задачи и примеры с модулями. Система линейных уравнений с. Математика 1 класс.13 3 Модуль Уравнения и неравенства с модулем: 9:. Тип урока: Урок обобщения понятия модуль с элементами самостоятельной работы. Целевая аудитория: для 6 класса. Уравнения с модулем в 6 классе сводятся к простейшим уравнениям, решение которых опирается на определение модуля. Решение неравенств с модулями. ИЗУЧЕНИЕ.

    МОДУЛЯ РАЦИОНАЛЬНОГО ЧИСЛА В 6 КЛАССЕ НА. Модуль и. Математика 1 класс. Математичний калькулятор. Решение уравнений с модулем 6 й класс. Как решать уравнения с модулем. Противоположные числа и модуль. Задачи в 6 классе. Третий интервал дает два корня, которые удовлетворяют исходное уравнение с модулями. Занятия по математике, русскому языку и литературе. Положительные и отрицательные числа. Сразу рассмотрим на примере решение уравнений. Уравнения с модулями. Модуль числа. Правила. Задания.

    С проверкой ответов. Материалы предыдущего фестиваля. Система линейных уравнений с модулями. Квадратные уравнения и системы уравнений. Получили простейшее уравнение с модулем. Квадратные уравнения с параметрами.6 часов Целью данного курса является формирование целостной системы решения упражнений с модулями и параметрами, формированию навыков организации учащимися самостоятельных микроисследований. При изучении данной темы мы опираемся на общепринятое определение модуля числа: Решение уравнений с модулем в 6 классе. Уравнением.

    С модулем абсолютной. Онлайн уроки для школьников с дистанционным репетитором через интернет. Решение уравнений с модулем. Математика 5 класса, повторение. Урок по математике для 6 класса. Определение модуля числа, примеры на модуль числа. Решить уравнение. Подготовка к контрольным, ЕГЭ. Форма заявки на участие. Понятно, что в этом случае уравнение не имеет решений, так как по определению модуль всегда неотрицателен. Категория: Презентации по математике.

     

    Вместе с Уравнения с модулями 6 класс часто ищут

     

    уравнения с модулем 7 класс

    модульные уравнения 6 класс

    решить уравнение с модулем онлайн калькулятор

    модуль числа 6 класс виленкин

    модуль числа 6 класс презентация

    неравенства с модулем примеры

    решение уравнений с модулем онлайн

    квадратные уравнения с модулем

     

    Читайте также:

     

    Скачать тесты по окружающему миру о. т поглазова 3 класс

     

    Решебник гдз по учебнику обшествоведенье 11 класс

     

    Задание по русскому языку на сегодня 3 класс 45 школа

     

    Графический способ решения уравнений с модулем

    Похожие презентации:

    Элементы комбинаторики ( 9-11 классы)

    Применение производной в науке и в жизни

    Проект по математике «Математика вокруг нас. Узоры и орнаменты на посуде»

    Знакомство детей с математическими знаками и монетами

    Тренажёр по математике «Собираем урожай». Счет в пределах 10

    Методы обработки экспериментальных данных

    Лекция 6. Корреляционный и регрессионный анализ

    Решение задач обязательной части ОГЭ по геометрии

    Дифференциальные уравнения

    Подготовка к ЕГЭ по математике. Базовый уровень Сложные задачи

    1. Тема урока

    Графический способ
    решения уравнений с модулем
    Тема урока

    2.

    Задай функцию формулой

    3. Задай функцию формулой

    4. Задай функцию формулой

    5. Задай функцию формулой

    6. Задай функцию формулой

    7. Задай функцию формулой

    8. Задай функцию формулой

    9. Задай функцию формулой

    10. Задай функцию формулой

    Выполнила ученица 10 «А»
    Засыпалова Анна

    17. Правило построения графика функции y=f(|x|). :

    1. Построим график функции y=f(x), для х≥0
    2. Достроим левую часть графика, симметричную
    построенной правой части относительно оси
    ординат

    18. 1.Построить график функции y=|x|

    1.Построить график функции
    Построение:
    1).
    х
    у
    0
    0
    2
    2
    Строим график функции
    y=x, х ≥0
    2)Симметрично
    отображаем
    построенную часть
    графика относительно
    оси оу.
    .
    y=|x|
    Y
    y=|x|
    X

    20. 2. Построить график функции Y=|x|²-4|x|+3

    2. Построить график функции
    +3
    1)Строим
    Y=|x|²-4|x|
    Построение.
    y=x²-4x+3,
    х≥0
    Y
    Y=|x|²-4|x|+3
    а)Хверш= -в/2а=-4/2=2
    Уверш=2²-4·2+3=-1
    (2;-1)-вершина параболы
    б)Нули функции (точки пересечения с ох)
    x²-4x+3=0
    х1=1
    х2=3
    (1;0) (3;0) — точки пересечения графика с осью ох
    в)Если х=0, то у=3
    (0;3)
    2).Симметрично отображаем построенную часть
    графика относительно оси оу
    X

    21. 3. Построить график функции у = sin|x|


    Y
    •X

    22. 4. Построить график функции y=cos|x|

    Y
    X

    23. 5. Построить график функции y= cos|x|+2

    5. Построить график функции y= cos|x|
    +2
    Y
    X

    24. 6. Построить график функции у=3|х|+2

    Построение.
    1)Cтроим график функции
    у=3х+2, х≥0
    х
    у
    0
    2
    Y
    1
    5
    X
    2).Симметрично
    отобразим построенную
    часть графика
    относительно оси ох.
    Y
    у=3|х|+2
    X
    Выполнила: Кузьмина Валерия
    ученица 10 «а» класса

    27. Правило Построения графика функции y=|f(x)|

    1. Построить график y=f(x)
    2.Сохранить без изменения части графика
    y=f(x), расположенные выше оси OX.
    3.Отобразить симметрично оси OX части
    графика y=f(x), расположенные ниже
    оси OX.
    y
    2
    -1
    0
    x

    30. 2) Построить график функции Y=|X²-4x+3|. Построение.

    y
    0
    1
    3
    x

    31. 3) Построить график функции y=|cosx| Построение.

    3) Построить график функции y=|
    cosx|
    Построение.
    y
    1
    0
    -1
    x

    32. 4)Построить график функции y=|sinx|-4 Построение.

    y
    1
    0
    -1
    -4
    x

    33. Решение уравнений, содержащих модуль

    Решение
    уравнений,
    содержащих
    Способы решения уравнений,
    содержащих модуль
    Использование
    определения модуля
    Графический способ
    Метод введения
    новой переменной

    35. Графический способ

    |х-2|=
    у
    х
    1. у =|х-2|
    2. у =
    0
    1 2
    4
    х
    Ответ: 1; 4.
    х

    36. Задания командам

    Реши уравнение графическим способом
    а) |x-1|=2;
    б) x2 = |x|;
    в)sin|x| = — х2;
    г) |x-1|= |x|-1;
    д) 3|x|=4 — х2 (МФТИ, 2000г)
    е)|x2-3x|=2x-4 (МГУ, 2000г)
    №/
    Команда
    1
    команда
    2
    команда
    1
    2
    3
    4
    5
    6
    Устная
    работа
    итог
    Слово «модуль» произошло от латинского слова «modulus», что в переводе
    означает «мера». Это многозначное слово(омоним), которое имеет множество
    значений и применяется не только в математике, но и в архитектуре, физике,
    технике, программировании и других точных науках.
    В архитектуре — это исходная единица измерения, устанавливаемая для данного
    архитектурного сооружения и служащая для выражения кратных соотношений его
    составных элементов.
    В технике — это термин, применяемый в различных областях техники, не имеющий
    универсального значения и служащий для обозначения различных коэффициентов
    и величин, например модуль зацепления, модуль упругости и .т.п.
    Модуль объемного сжатия( в физике)-отношение нормального напряжения в
    материале к относительному удлинению.

    44. Модуль числа

    Абсолютная величина или модуль,
    обозначается |x|, |x-1| , |a|

    45. Знак модуля

    Считают, что термин предложил
    использовать
    Котс, ученик Ньютона. Лейбниц тоже
    использовал
    эту функцию, которую называл модулем и
    обозначал: mol x.
    Общепринятое обозначение
    абсолютной величины введено в 1841
    году Вейерштрассом.
    Для комплексных чисел это понятие ввели
    Коши и Арган в начале XIX века.
    Математики шутят.
    Трехмерная кубическая линейка
    Калькулятор для умных

    English     Русский Правила

    Модуль в линейных уравнениях с двумя переменными

    • Выражение
    • Уравнение
    • Неравенство
    • Свяжитесь с нами
    • Упрощение
    • Факт или
    • Expand
    • GCF
    • LCM
    • Решить
    • График
    • Система
    • Решение
    • График
    • Система
    • Математический решатель на вашем сайте

    Наших пользователей:

    Если у вас нет денег, чтобы платить домашнему репетитору, тогда Алгебратор — это то, что вам нужно, и поверьте мне, он делает все, что репетитор сделал бы, и, возможно, даже больше.
    Дэниел Коттон, Невада

    Я действительно боролся со старой версией. .. настолько сильно, что просто отказался от нее. Эта новая версия выглядит лучше и кажется более удобной для навигации. Думаю будет здорово! Спасибо!
    Том Сэнди, NE

    Я использовал вашу программу для подготовки к экзамену по алгебре. Мне очень нравится пошаговый процесс решения и пояснения.
    Ричард Уильямс, Лос-Анджелес

    Нет проблем, эта новая программа очень проста в использовании и понимании. Это хорошая программа, я желаю вам всего наилучшего. Спасибо!
    Мэри Браун, Северная Дакота

    Я работаю учителем математики в старшей школе более шестнадцати лет, и, к счастью, со времен Apple II у меня были компьютеры, которые помогали мне в классе. Но ничто еще не дало результатов и не помогло моим ученикам понять столько сложных уравнений и понятий, как Алгебратор! Вот почему в настоящее время я слежу за тем, чтобы каждая школьная система (даже ноутбуки) работала на ней! По крайней мере, это делает нашу работу намного проще!
    Адам Боттс, Флорида


    Студенты, борющиеся со всевозможными задачами по алгебре, узнают, что наше программное обеспечение спасает им жизнь.

    Вот поисковые фразы, которые сегодняшние поисковики использовали, чтобы найти наш сайт. Сможете ли вы найти среди них свою?
    Поисковые фразы, использованные 04.11.2009:
    • Калькулятор стандартной формы онлайн
    • задания 3 класс чтение 2007 рабочие листы
    • «разложить следующие выражения методом общих множителей»
    • ПРОЦЕНТНЫЕ ФОРМУЛЫ
    • обучение алгебре онлайн
    • онлайн-рабочих листа + поиск тома
    • образец бумаги 7 класса
    • решить онлайн частное рациональных выражений
    • Однородные непостоянные коэффициенты 2-го порядка
    • Калькулятор умножения и упрощения рациональных выражений
    • Рабочие листы переменных деления дробей
    • три фута на четыре фута равно количеству дюймов
    • Таблица формул GED
    • онлайн калькулятор для решения неравенств
    • математический лист для комбинаций и перестановок
    • прентис холл математика алгебра 1 онлайн книга
    • рабочие листы по наибольшему общему делителю
    • ответ по алгебре 1
    • практика перестановки
    • факторные уравнения онлайн
    • алгебраизатор
    • калькулятор упрощающих подкоренных выражений
    • как разложить трехчлен куба на множители
    • завершить квадратный калькулятор
    • квадратный корень с использованием коэффициентов
    • алгебра пицца
    • решение задач и пропорции с рациональными выражениями рабочие листы
    • как решить ряд по математике
    • классная математика для детей. com
    • Вычисление квадрата и умножения
    • Рабочие листы пересечения уклонов
    • проверить приведенное уравнение -3-4(t-5)=2(t+3)+11
    • упорядочивание дробей от наибольшего к наименьшему листу
    • excel полином 3-го порядка
    • практика Пирсона 12-4 ответы алгебра 1 помощь
    • Бесплатное домашнее задание по первому курсу дифференциальных уравнений с приложениями для моделирования
    • листа ответов для Макдугала Литтелла
    • бесплатные учебные листы по математике для 9-го класса для квадратных корней
    • листы деления целых чисел
    • Пример мировой задачи на показатели с решением или ответами
    • слабое решение уравнения в частных производных первого порядка
    • как возвести в квадрат многочлен в знаменателе
    • prentice hall сложение экспонент
    • математические углы
    • учебник по математике для 1 класса (Онтарио)
    • математика для чайников
    • абсолютные значения и радикалы
    • программирование Ti-83 плюс делать sin cos tan
    • планы уроков по уравнениям с двумя переменными
    • рабочих листа по математике, которые вы можете сделать онлайн
    • логарифм домашнее задание ответы
    • вычитание в слове
    • распечатки по математике практика
    • Нахождение общего знаменателя в полиномиальных уравнениях
    • Образец вступительного экзамена по математике для 9-го класса
    • целая сумма Java
    • как решить мнимые числа в ti 84
    • Программа решения кубических уравнений TI-83
    • математических анкеты для второго года обучения
    • Курс математики 1 ответы
    • базовая алгебра для четвероклассников
    • онлайн-решатель факторинга
    • Поиск слов по истории Техаса 7-го класса
    • Урок PowerPoint о сложении и вычитании
    • Как сделать элементарную алгебру
    • калькулятор решения алгебраических выражений бесплатно
    • вероятности на ti 83
    • написание программ калькулятора факторинга
    • «Распределительное свойство алгебры»
    • Алгебра Гленко 1 ответы
    • бесплатных математических листа для ged
    • алгебра 2 с pizazz
    • Дифференциальные уравнения цепей первого порядка
    • алгебра 1 онлайн для чайников
    • ПРОСТАЯ ЗАДАЧА С ИНТЕРЕСОМ + решатель
    • решение квадратных уравнений, представляющих собой последовательные целые числа
    • f(x) квадратный корень из дроби
    • математический в Visual Basic
    • google элементарная математика нечетная дробь
    • бесплатный калькулятор квадратного корня, где я могу ввести свои ответы в
    • Алгебра с ключом ответов Pizzazz
    • решение уравнений с несколькими корнями с помощью Excel
    • уроки дроби в четвертом классе
    • график уравнения лайнера
    • как обучать детей технической алгебре
    • калькулятор упрощения произведения суммы
    • бесплатные оценки для восьмиклассников
    • Рабочий лист графика наклона/пересечения
    • Алгебра 1/ Тригонометрия регентов
    • факторинг трехчленов 9 класс математика
    Предыдущий Далее

    Квадратное уравнение Python — javatpoint

    следующий → ← предыдущая

    Квадратное уравнение:

    Квадратное уравнение образовано от латинского термина «quadrates», что означает квадрат. Это особый тип уравнения, имеющий форму:

    топор 2 +bx+c=0

    Здесь «x» неизвестен, который нужно найти, а «a», «b», «c» задает такие числа, что «a» не равно 0. Если a = 0, то уравнение становится линейным, а не квадратичным больше.

    В уравнении a, b и c называются коэффициентами.

    Возьмем пример решения квадратного уравнения 8x 2 + 16x + 8 = 0

    См. этот пример:

    # импортировать сложный математический модуль импортировать cmath a = float(input(‘Введите a:’)) b = float(input(‘Введите b:’)) c = float(input(‘Введите c:’)) # вычисляем дискриминант д = (б**2) — (4*а*в) # найти два решения sol1 = (-b-cmath.sqrt(d))/(2*a) sol2 = (-b+cmath.sqrt(d))/(2*a) print(‘Решение: {0} и {1}’.format(sol1,sol2))

    Вывод:

     Введите: 8
    Введите б: 5
    Введите с: 9
    Решение: (-0,3125-1,0135796712641785j) и (-0,3125+1,0135796712641785j)
     

    Пояснение —

    В первой строке мы импортировали модуль cmath и определили три переменные с именами a, b и c, которые принимают ввод от пользователя. Затем вычисляем дискриминант по формуле. Используя метод cmath.sqrt() , мы вычислили два решения и распечатали результат.

    Второй метод

    Мы можем получить решение квадратного уравнения, используя прямую формулу. Давайте разберем следующий пример.

    Вышеприведенная формула состоит из следующих случаев.

    • Если b 2 < 4ac, то корни комплексные (не вещественные). Например - х 2 + х + 1, корни -0,5 + i1,73205 и +0,5 — i1,73205.
    • Если b 2 == 4ac, то оба корня одинаковы. Например — x 2 + x + 1, корни равны -0,5 + i1,73205 и +0,5 — i1,73205.
    • Если b 2 > 4ac, то корни действительны и различны. Например — х 2 — 7 х — 12, корни 3 и 4.

    Пример —

    # Программа Python для поиска корней квадратного уравнения импортировать математику # функция поиска корней def findRoots(a, b, c): dis_form = б * б — 4 * а * с sqrt_val = math. sqrt (abs (dis_form)) если dis_form > 0: print(«настоящие и разные корни») print((-b + sqrt_val) / (2 * a)) print((-b — sqrt_val) / (2 * a)) Элиф dis_form == 0: print(» действительные и одинаковые корни») напечатать (-b / (2 * а)) еще: print(«Сложные корни») print(- b / (2 * a), » + i», sqrt_val) print(- b / (2 * a), «- i», sqrt_val) a = int(input(‘Введите a:’)) b = int(input(‘Введите b:’)) c = int(input(‘Введите c:’)) # Если a равно 0, то неверное уравнение если а == 0: print(«Введите правильное квадратное уравнение») еще: найтиКорни (а, б, в)

    Вывод:

     Введите a:7
    Введите б:5
    Введите c:2
    Сложные корни
    -0,35714285714285715 + i 5,5677643628300215
    -0,35714285714285715 - я 5,5677643628300215
     

    Пояснение —

    В приведенном выше коде мы импортировали математический модуль и определили формулу для вычисления дискриминанта.

    Примеры подмножеств и множеств: Множества и подмножества, число, мощность, примеры, урок и презентация по алгебре, 9 класс

    Примеры решений

    К главной странице


    Тема: множества. Задача. Докажите, что множество $A$ тогда и только тогда является подмножеством множества $B$, когда каждый элемент, не принадлежащий $B$, не принадлежит $A$. Решение. Оборот «тогда и только тогда» означает два утверждения: 1) $(A \subset B) \Rightarrow {}$(для любого $x$, не принадлежащего $B$, $x$ не принадлежит $A$), 2) (для любого $x$, не принадлежащего $B$, $x$ не принадлежит $A$)${} \Rightarrow (A \subset B)$. Продемонстрируем на примере этой задачи, как применяется метод доказательства «от противного». Чтобы доказать некоторое утверждение, мы предполагаем, что оно не выполняется и приходим к противоречию Докажем здесь только утверждение 1). Предположим противное, то есть что $A \subset B$, но существует некоторый элемент $x$, не принадлежащий $B$, но принадлежащий $A$. По определению того, что $A \subset B$ мы знаем, что каждый элемент, принадлежащий $A$, должен принадлежать $B$. В частности, $x \in B$. Возникает противоречие. Значит, наше предположение было неверно, и нет элементов принадлежащих $A$, но не принадлежащих $B$. Комментарий. Заметим, что эта внешне несложная задача является одновременно очень важной и достаточно сложной. Как и в некоторых других задачах листка «Множества», зачастую не совсем понятно, что, собственно, надо доказывать. Мы одновременно демонстрируем метод от противного, учимся писать отрицание к следствию и разбираемся, что значит «тогда и только тогда».


    Тема: множества. Задача. Может ли у множества быть ровно: 1) 0; 2) 7; 3) 16 подмножеств? Решение. Во-первых, заметим, что число подмножеств пустого множества равно 1, число подмножеств множества, состоящего из одного элемента, равно 2, из двух – 4, из трех – 8, из четырех – 16. Второе важное наблюдение – монотонность: если у одного множества больше элементов, чем у другого, то и подмножеств у него больше. {-1}[B_1\cup B_2]$.


    Тема: Комбинаторика. Задача. Сколько существует «cлов» из двух букв английского языка? Комментарий. Во-первых, в этой задаче, конечно, имеются в виду не те слова, которые можно встретить в словаре, а произвольные сочетания букв английского языка. Теперь перейдем к главным идеям решений подобных комбинаторных задач. Когда требуется посчитать кол-во каких-то специальных объектов, часто работает следующая схема: 1. перечисление всех вариантов в разумном порядке; 2. понимание того, какие «записи» соответствуют одному и тому же объекту. В этой задаче нам понадобится только 1. Второй пункт будет проиллюстрирован в следующем примере. Другая полезная (отнюдь не только в комбинаторике) идея состоит в том, что, прежде чем решать задачу для произвольного (или очень большого) $n$, полезно сначала разобраться со случаем небольших $n$. Замеченные при этом закономерности (даже если они не доказаны) часто существенно упрощают решение задачи. Решение. Попробуем для начала решить ту же задачу, но с алфавитом, состоящим всего из трех букв – а, b, c. Здесь мы можем явно выписать все слова, состоящие из двух букв: аа, аb, аc, bа, bb, bc, cа, cb, cc. Получили $9=3\cdot3$. В общем случае подсчет числа таких слов будем вести следующим образом: сначала посчитаем число слов, которые начинаются с буквы «а», затем – число слов, начинающихся с буквы «b» и так далее. Совершенно очевидно, что таким образом мы посчитаем каждое «слово» по разу. Заметим, что двухбуквенных слов, начинающихся на букву «а», будет ровно столько же, сколько есть букв в алфавите. Слов, начинающихся на букву «б», будет столько же. И так со всеми буквами. В нашей задаче слов, начинающихся с буквы «а», будет 26. Столько же будет слов, начинающихся с буквы «b», и так далее. Значит, всего слов, состоящих из двух букв, будет $26\cdot 26=676$. Комментарий. На самом деле тут речь идет о прямом (или декартовом) произведении множеств. Произведением множеств называют множество из упорядоченных пар. Число элементов в произведении множеств равно произведению числа элементов исходных множеств (правило умножения). В нашем случае множество всех слов из 2 букв на английском языке – это как раз множество неупорядоченных пар английских букв, а значит равно произведению множества букв английского алфавита (их 26 штук) с самим собой, а значит в нем 26*26=676 элементов.


    Тема: комбинатрика. Задача. Сколькими способами можно выбрать из десяти человек двух дежурных и одного старшего дежурного? Решение. Полезным упражнением является честное выписывание троек дежурных, выбираемых из, скажем, четырех человек (Вася, Коля, Леша, Петя). При этом первым будем писать старшего дежурного, а остальных дежурных записывать в алфавитном порядке. Удобно при этом сначала выбирать старшим дежурным первого по алфавиту, потом второго по алфавиту и т.д. Так как порядок «обычных» дежурных не важен, выписывая их по алфавиту, мы никакой случай не забудем и не повторим дважды. (Вася, Коля, Леша), (Вася, Коля, Петя),& (Вася, Леша, Петя), (Коля, Вася, Леша), (Коля, Вася, Петя),& (Коля, Леша, Петя), (Леша, Вася, Коля), (Леша, Вася, Петя),& (Леша, Коля, Петя), (Петя, Вася, Коля), (Петя, Вася, Леша),& (Петя, Коля, Леша). Итого $12$ вариантов. Теперь перейдем к решению исходной задачи. Старшего дежурного можно выбрать десятью способами. Когда мы уже выбрали старшего, мы можем выбрать первого дежурного девятью (потому что одного человека мы уже назначили старшим дежурным) способами. После того, как мы выбрали старшего дежурного и еще одного дежурного, второго дежурного мы можем выбрать восемью способами. Итого, вроде бы, по «правилу умножения» должно получаться $10\cdot9\cdot8=720$. Однако если мы внимательно посмотрим на наше решение, то увидим, что каждую тройку дежурных мы посчитали по два раза. Например, тройку дежурных (Вася – старший дежурный, Леша, Петя) мы посчитали два раза: как тройку (Вася, Петя, Леша) и как тройку (Вася, Леша, Петя). Поэтому правильный ответ в два раза меньше – 360.


    Подмножества

    Если каждый элемент множества В является в то же время элементом множества А, то говорят, что В – подмножество в А, и пишут . Каждое непустое множество имеет по крайней мере два подмножества: пустое множество Ø и само множество А. Таким образом, пустое множество является подмножеством любого множества.

    Приведем примеры подмножеств:

    1. числовой отрезок [–1, 3] есть подмножество числового отрезка [–4; 5];

    2. множество всех квадратов есть подмножество множества всех прямоугольников;

    3. множество Z всех целых чисел есть подмножество множества Q всех рациональных чисел;

    4. множество точек треугольника, вписанного в круг, есть подмножество множества точек этого круга;

    5. множество точек круга является подмножеством множества точек описанного вокруг него квадрата;

    6. множество звезд нашей Галактики является подмножеством множества всех звезд Вселенной;

    7. множество учеников восьмого класса данной школы есть подмножество множества всех учеников этой школы. В свою очередь множество учеников этой школы является подмножеством множества всех школьников в Республики Беларусь;

    8. множество жителей Москвы является подмножеством множества всех жителей России;

    9. множество граждан г. Минск является подмножеством множества всех людей на земном шаре.

    Н а рисунке 6 прямые MN и PQ пересекаются в точке R.

    Эта точка принадлежит как прямой MN, так и прямой PQ, т.е. является общим элементом двух множеств – множества точек прямой MN и множества точек прямой PQ. Точно так же множество точек прямой MN и множество точек окружности Г (рис. 7) имеют два общих элемента – точку А и точку B. Прямая и окружность пересекаются в двух точках.

    Введем теперь общее понятие пересечения нескольких множеств. Пересечением множеств А и В называют новое множество Х, содержащее те и только те элементы, которые входят и в множество А и в множество В.

    Пересечение множеств А и В обозначают или AB. Например, если А – множество мальчиков, обучающихся в данной школе, а В – множество всех учеников из 8 класса, то – множество мальчиков, которые учатся в 8 классе.

    С понятием пересечения множеств приходится иметь дело и в арифметике. Пусть А – множество натуральных делителей числа 72:

    A ={1, 2, 3, 4, 6, 8, 9, 12, 18, 24, 36, 72},

    а В – множество натуральных делителей числа 54:

    B ={1, 2, 3, 6, 9, 18, 27, 54}.

    Тогда множество состоит из чисел 1, 2, 3, 6, 9, 18:

    ={1, 2, 3, 6, 9, 18}.

    Эти числа являются общими делителями для 72 и 54. Наибольший элемент множества равен 18. Это – наибольший общий делитель чисел 54 и 72. Множество делителей числа 72 конечно. А множество кратных этого числа бесконечно:

    С = {72, 144, 216, … , 72n, …}.

    Бесконечно и множество кратных числа 54:

    D = {54, 108, 162, 216, … , 54m, …}.

    Пересечением этих множеств является множество общих кратных для чисел 72 и 54:

    = {216, 432, …}.

    Наименьшее число в , т.е. 216, называется наименьшим общим кратным для 72 и 54.

    Иногда приходится пересекать множества геометрических фигур. Например, множество всех квадратов является пересечением множества всех прямоугольников с множеством всех ромбов, т.к. квадрат – это фигура, являющаяся одновременно и прямоугольником, и ромбом. Пересечением множества всех треугольников с множеством всех правильных многоугольников является множество правильных треугольников.

    Знакомство с множествами и подмножествами

    В сегодняшней статье мы рассмотрим небольшое введение в множества и подмножества: какие существуют типы, как они выражаются… Все с некоторыми примерами, которые очень легко понять. Для этого вместо чисел мы будем использовать такие элементы, как фрукты, животные, дети…

    Первое, что мы должны знать, это , что такое множество.  Мы можем определить его как набор объектов, который мы называем элементами , которые имеют некоторые общие характеристики.

    Наборы могут содержать элементы любого типа: цифры, буквы, предметы, люди… Например, этот набор содержит фрукты:

    НАБОР

    Классификация наборов:

    Набор можно классифицировать по в количество элементов, которые он имеет. Это:

    • Конечное: , если у него есть коллекция, которую можно подсчитать, даже если это может быть трудно сделать. Например, набор фруктов включает в себя все виды фруктов, которые существуют в мире. Несмотря на то, что это может быть трудно, можно сосчитать все виды фруктов в мире, поэтому оно конечно.
    • Бесконечно: , если у него есть коллекция, которая никогда не может закончить подсчет. Например, множество всех четных чисел (которые бесконечны) является бесконечным множеством.

    Отношения между множествами:

    С точки зрения отношения между ними множества могут быть:

    • Непересекающиеся множества: те множества, которые не имеют ни одного общего элемента.

    Например, множества фруктов и животных не пересекаются, потому что нет ни плода, который мог бы быть животным, ни животного, которое могло бы быть плодом:

    РАЗЪЕДИНЕННЫЕ МНОЖЕСТВА

    • Равные множества:   бывает, когда все элементы одного множества принадлежат другому.

    Например, набор красных фруктов и набор желтых фруктов являются равными наборами фруктов, поскольку все красные фрукты являются фруктами, и все желтые фрукты также являются фруктами.

    ПОДМНОЖЕСТВА

    Множество живых существ очень велико: оно имеет множество подмножеств. Например:

    • Растения — это подмножество живых существ.
    • Животные — это подмножество живых существ.
    • Люди являются подмножеством животных.

    Пересечение подмножеств: иногда разные наборы отличаются друг от друга, но имеют некоторые общие элементы. Затем определяется область пересечения, содержащая все общие элементы.

    Например, у нас есть группа девушек и группа людей, которые носят очки. Поскольку есть девушки, которые носят очки, они составляют часть пересечения двух наборов:

    ПЕРЕКРЕСТОК

    Что вы думаете об этом посте? Помогло ли это вам лучше понять множества и подмножества?

    Если вы хотите попрактиковаться в этих наборах упражнений и многом другом, зарегистрируйтесь и попробуйте бесплатную пробную версию Smartick.

    Подробнее:

    • Автор
    • Последние сообщения

    Smartick

    Команда создания контента.
    Мультидисциплинарная и мультикультурная команда, состоящая из математиков, учителей, профессоров и других специалистов в области образования!
    Они стремятся создать наилучший математический контент.

    Последние сообщения Smartick (посмотреть все)

    Урок по подмножествам | Математические вкусности

    Форма поиска

    Поиск

    Пример 1: Учитывая  A  = {1, 2, 4} и B  = {1, 2, 3, 4, 5}, какова связь между этими множествами?

    Мы говорим, что A  является подмножеством B , так как каждый элемент A также находится в B . Это обозначается:

    Диаграмма Венна для отношения между этими множествами показана справа.

     

    Ответ: A  является подмножеством B .


    Другой способ определения подмножества: A  является подмножеством B  если каждый элемент A  содержится в Б . Оба определения показаны на приведенной выше диаграмме Венна.

    Пример 2: Учитывая  X  = {a, r, e} и Y  = {r, e, a, d}, какова связь между этими множествами?

    Мы говорим, что X  является подмножеством Y , поскольку каждый элемент X также находится в Y.  Обозначается:

    Диаграмма Венна для связь между этими наборами показана справа.

     

    Ответ: X  является подмножеством Y .


    Пример 3: Учитывая  P  = {1, 3, 4} и Q  = {2, 3, 4, 5, 6}, какова связь между этими множествами?

    Мы говорим, что P не является подмножеством Q s , поскольку не каждый элемент P не содержится в 90 008 Q.  Например, мы можем видеть, что 1  Q . Утверждение  «P не является подмножеством Q»  обозначается:

    Обратите внимание, что эти множества имеют некоторые общие элементы. Пересечение этих множеств показано на диаграмме Венна ниже.

    Ответ: P не является подмножеством Q .


    Обозначения для подмножеств показаны ниже.

    Символ Значение
    является подмножеством
    не является подмножеством

    Пример 4: Даны A  = {1, 2, 3, 4, 5} и B  = {3, 1, 2, 5, 4}, какая связь между A и  B ?

    Анализ: Напомним, что порядок, в котором элементы появляются в наборе, не важен. Глядя на элементы этих наборов, становится ясно, что:

    Ответ: A и B эквивалентны.


    Определение: Для любых двух наборов, если A B и B A, , то 9 0008  A = B. Таким образом,  A  и  B  эквивалентны.

    Пример 5: Список всех подмножеств набора C  = {1, 2, 3}.

    Ответ:

    Подмножество Список всех возможных комбинаций элементов
    Д  = {1} по одному
    E  = {2} по одному
    Ф  = {3} по одному
    G  = {1, 2} два за раз
    M  = {1, 3} два за раз
    N  = {2, 3} два за раз
    P  = {1, 2, 3} три за раз
    Ø Пустой набор не содержит элементов.

    Глядя на пример 5, вы можете удивиться, почему нулевой набор указан как подмножество C. В нулевом наборе нет элементов, поэтому в нулевом наборе не может быть элементов, которые не являются содержится в комплекте. Следовательно, нулевой набор является подмножеством каждого набора.  Вам также может быть интересно: является ли множество подмножеством самого себя?  Ответ положительный: любое множество содержит себя как подмножество. Это обозначается как:

    A  A.

    Подмножество, которое меньше полного набора, называется правильным подмножеством . Таким образом, набор {1, 2} является правильным подмножеством набора {1, 2, 3}, поскольку элемент 3 не входит в первый набор. В примере 5 видно, что G является правильным подмножеством C . На самом деле каждое подмножество, указанное в примере 5, является правильным подмножеством C, , кроме P . Это связано с тем, что P и C являются эквивалентными множествами ( P  = C ) . Некоторые математики используют символ  для обозначения подмножества и символ  для обозначения правильного подмножества, с определением для правильных подмножеств следующим образом:

    If A    B и A   B , тогда A называется собственным подмножеством B и обозначается A B .

    Несмотря на то, что указанная выше информация важна, она может немного запутать. Итак, давайте подумаем о подмножествах и надлежащих подмножествах следующим образом:

    Подмножества и правильные подмножества
    Множество {1, 2} является правильным подмножеством множества {1, 2, 3}.
    Множество {1, 2, 3} не является правильным подмножеством множества {1, 2, 3}.

    Видите ли вы закономерность в приведенных ниже примерах?

    Пример 6: Список всех подмножеств набора R  = {x, y, z}. Сколько их там?

    Подмножества
    D  = {x}
    E  = {г}
    F  = {z}
    G  = {х, у}
    Н  = {х, г}
    J  = {у, г}
    К  = {х, у, г}
    Ø

    Ответ: Существует восемь подмножеств множества R  = {x, y, z}.


    Пример 7: Список всех подмножеств набора C  = {1, 2, 3, 4}. Сколько их там?

    Подмножества
    Д  = {1} М  = {2, 4}
    E  = {2} N  = {3, 4}
    Ф  = {3} О  = {1, 2, 3}
    Г  = {4} P  = {1, 2, 4}
    H  = {1, 2} Q  = {1, 3, 4}
    J  = {1, 3} Ч  = {2, 3, 4}
    К  = {1, 4} S  = {1, 2, 3, 4}
    Л = {2, 3} Ø

    Ответ: Существует 16 подмножеств множества C  = {1, 2, 3, 4}.


    В примере 6 набор R состоит из трех (3) элементов и восьми (8) подмножеств. В примере 7 набор C состоит из четырех (4) элементов и 16 подмножеств. Чтобы найти количество подмножеств множества из n элементов, возведите 2 в n-ю степень: То есть:

    Количество подмножеств в наборе A равно 2 n , где n — количество элементов в наборе A.


    Итоги урока 9000 3

    Подмножество:  A  есть подмножество B: , если каждый элемент A содержится в B . Обозначается A B.

    Эквивалентные наборы: Для любых двух наборов, если A  B  и     B  A, , затем  A = B.

    Нулевой набор: Нулевой набор является подмножеством каждого набора .

    Наборы и подмножества:  Любой набор содержит себя как подмножество. Это обозначается A A.

    Правильные подмножества: If A   B и A  9000 8 ≠   B , тогда A называется правильным подмножеством Б  и обозначается A    B .

    Количество подмножеств: Количество подмножеств в наборе A равно 2 n , где n — количество элементов в наборе A.


    Упражнения 9002 5

    Указания: Прочитайте каждый вопрос ниже. Выберите свой ответ, нажав на его кнопку. Обратная связь по вашему ответу представлена ​​в ОКНО РЕЗУЛЬТАТЫ. Если вы допустили ошибку, обдумайте свой ответ еще раз, а затем выберите другую кнопку.

    1. Что из следующего является подмножеством множества G?

     G = {d, a, r, e}
     

      X = {e, a, r}
      Y = {e, r, a}
      Z = {r, e, d}
     Все вышеперечисленное.

    ОКНО РЕЗУЛЬТАТОВ:
     

    2. Какое из следующих утверждений верно?
     
      {гласные}   {согласные}
     {согласные}    {гласные}
     {гласные}    {алфавит}
     Ничего из вышеперечисленного.

    ОКНО РЕЗУЛЬТАТОВ:
     

    3. Что из нижеперечисленного НЕ является подмножеством набора A?

    А = {2, 3, 5, 7, 11}


     
      B  = {3, 5, 2, 7}
      C = {2, 3, 7, 9}
      = {7, 2, 3, 11}
     Все вышеперечисленное.

    ОКНО РЕЗУЛЬТАТОВ:
     

    4.

    Графики 3d: Построить трехмерный график онлайн

    Что такое 3D-графика – База знаний Timeweb Community

    Под понятие 3D-графики можно отнести двухмерные изображения с элементами объема, который придается за счет работы с освещением и другими элементами, создающими на экране визуальную иллюзию. Еще к 3D-графике относятся полноценные трехмерные модели, создаваемые в специальных программах и применяемые в играх, кинематографе и мультипликации.

    Далее я предлагаю детальнее остановиться на этом типе графики, разобраться во всех ее тонкостях, характеристиках и принципах создания при помощи современных технологий.

    Что такое 3D-изображение

    Для начала остановимся на 3D-изображениях и поймем, что вообще делает их трехмерными и какие типы картинок можно отнести к этой категории. Если при просмотре изображения вы можете описать ширину и высоту, но не наблюдаете глубины, значит, это двухмерная графика. Значки на рабочем столе и указатели на улицах – все это относится к 2D-графике (за некоторым исключением, когда художник использует тень или другие приемы, чтобы сделать картинку объемной). 3D-изображение обязательно обладает глубиной, то есть является объемным. Простой пример такой графики вы видите на следующем изображении:

    Если нарисовать квадрат, представив только основные его четыре линии, это будет двухмерная модель. Но если немного повернуть квадрат, дорисовать грани и вершины, получится куб, являющийся объемным элементом, а значит, к нему относится характеристика 3D-модели. 

    Комьюнити теперь в Телеграм

    Подпишитесь и будьте в курсе последних IT-новостей

    Подписаться

    История развития 3D

    Полноценное представление 3D-элементов на экране мир увидел в короткометражном фильме «A Computer Animated Hand», вышедшем в 1972 году. На скриншоте ниже вы видите то, как аниматоры смогли спроектировать человеческую руку и анимировать ее на экране.

    Это дало сильный толчок в развитии анимационных технологий и применении подобных эффектов в кинематографе. Одним из первых фильмов, в котором зритель мог увидеть анимацию человеческого лица, считается «Futureworld», вышедший в 1976 году. Сразу после этого трехмерная графика начала прогрессировать очень быстро. Появились специальные программы, кинокомпании стали набирать сотрудников соответствующих должностей и реализовывали самые разные эффекты в своих проектах. Обладатели персональных компьютеров уже в начале 80-х годов могли скачать программу под названием 3D Art Graphics, которая включала в себя набор различных трехмерных объектов и эффектов. 

    Создание трехмерной графики

    Как же работает трехмерная графика на компьютерах и на какие этапы делится ее создание? 

    • 3D-моделирование. На компьютере создается модель, в точности передающая форму объекта, который нужно представить. Это может быть любой предмет, животное или человек. В общем, все, что нас окружает. Существует несколько видов трехмерного моделирования, каждый из которых имеет свои особенности и принципы, но сейчас не будем вдаваться в эту тему. Если хотите, можете ознакомиться с такими программами, как Blender или 3Ds Max, чтобы узнать, как трехмерные объекты рисуются при помощи программ.

    • Сценарий и анимация. Модели всегда размещены на сцене и необходимы для выполнения определенного действия: перемещения, разрушения или передачи любого другого эффекта. Для расположения объектов на сцене и их анимирования может использоваться та же программа, которая применялась и для моделирования, но иногда разработчики обращаются к другому софту. Анимации тоже бывают разными, например, сейчас особо популярен захват движения (когда программа считывает движения человека и передает их на трехмерную фигуру).

    • Рендеринг. Завершающий процесс работы над проектом. Подразумевает обработку цветов, типов поверхности, освещения и всех других параметров сцены. Для обработки необходим мощный компьютер, способный быстро считывать кадры и выдавать на экран необходимый результат.

    3D-моделирование

    В рамках этой статьи остановимся только на 3D-моделировании, поскольку именно этот процесс и является основной трехмерной графики. Вы уже знаете, что для выполнения данной операции используется специальный софт. Аниматор может взаимодействовать как с отдельными геометрическими фигурами и точками, преобразовывая их в необходимый объект, так и с одной болванкой, доводя ее до необходимой формы (как скульптор в реальной жизни).

    Изначально модель имеет серый цвет, поэтому обязательным этапом является наложение текстур и материалов. В крупных компаниях этим занимается специально обученный человек, получивший заготовку от 3D-моделировщика. Он по эскизам или специальным шаблонам накладывает на модель различные элементы, имитирующие волосы, ткань или типы поверхностей. Это и делает 3D-модель похожей на настоящую.

    Тему можно развивать бесконечно, поскольку 3D-графика обладает огромным множеством интересных особенностей, которые делают индустрию такой сложной и высокооплачиваемой. Кинокомпании тратят миллионы долларов на создание моделей и эффектов, которые в реальной жизни повторить проблематично и еще более затратно. Сейчас при помощи 3D-графики создаются практически все современные игры и мультфильмы.

    3D-графика: актуальность, направления и мнение эксперта

    Каждый из нас при работе за компьютером взаимодействует с трехмерной графикой. Обычно мы просто не обращаем на это внимания: идеальные элементы оформления, анимированные изображения и 3D-модели – это уже привычные составляющие рекламы и Интернет-приложений.

    Прежде чем построить дом или разработать дизайн интерьера – необходимо создать проект. Когда-то это было трудным и кропотливым делом, а сегодня благодаря 3D-графике это делают быстрее и качественнее.

    Сферы применения 3D-графики

    3D моделирование крепко вошло в современную жизнь и уже применяется в рекламе, архитектуре, индустрии. После появления 3D-печати, трехмерное моделирование перешло на новый уровень и стало еще более популярным. Соответственно, появилась новая профессия: 3D-дизайнер, имеющий множество специализаций.

    ONLINE курс польского языка

    Регистрируйся на авторский курс от UniverPL
    Выбирай удобное время — мы подстроимся под ваш график.

    Забронировать место

    Где сейчас используется 3D-графика:

    • Мультипликация: создание мультфильмов;
    • Создание компьютерных игр – разработка 3D контента для игр;
    • 3D визуализация архитектуры. Можно создать ролик, демонстрирующий все проектируемые этажи здания до начала строительства;
    • Промышленность: сложные детали собирают по готовой 3D модели;
    • Медицинская сфера;
    • Реклама и маркетинг.

    Направления и преимущества изучения 3D-графики

    Быстрое развитие 3D моделирования предполагает возникновение множества направлений, в которых можно развиваться. Основными являются:

    • Визуализация интерьеров: разработка интерьера и экстерьера до начала ремонта;
    • Визуализация фасадов: разработка фасада дома до начала строительства;
    • Моделирование ландшафтов;
    • Анимация;
    • 3D печать;
    • Визуальные (VFX) эффекты, применяемые в кинопроизводстве;
    • GameDev – разработка компьютерных игр;
    • Game design – дизайн компьютерных игр.

    К тому же, каждое направление имеет еще массу специализаций.

    Сегодня на основе трехмерной графики создают точные копии объекта, воплощают в жизнь некогда нереальные дизайнерские задумки. 3D технологии приносят пользу человечеству и большую прибыль специалистам сферы. Начать изучение 3D-моделирования в университете – это сделать старт в профессии. На момент выпуска из института вы будете на достаточном уровне, чтобы стать необходимым для рынка молодым специалистом.

    В Польше есть два университета, которые предоставляют качественные и актуальные знания по 3D-графике, особенно по разработке компьютерных игр. Это Польско-Японская Академия Компьютерных Технологий в Варшаве и Нижнесилезский Университет во Вроцлаве.

    Специальность 3D-графика для тех, кто интересуется индустрией игровых разработок. Студенты получают теоретические и практические знания, связанные как с технологической ИТ-подготовкой, так и в сфере дизайна и креативных индустрий. Занятия проводят эксперты, работающие над лучшими польскими мобильными играми. Стоит отметить, что в Польше расположены такие компании как 11 bit studios S.A., CD Projekt RED, которые выпускают известные миру компьютерные игры. Поэтому проблем с трудоустройством после успешного окончания университета у выпускников нет.

    Что важно знать о компьютерной графике? Как получить работу в этой сфере и какое направление выбрать?

    На эти вопросы нам ответил Oleksii Havrysh, имеющий 6 лет опыта в GameDev, он работал на таких проектах как “Halo Infinite” и “World of Tanks”, а сейчас занимает должность Level/Environment Artist.

    Как начался ваш путь в 3D-графике и почему выбрали сферу компьютерных игр?

    Я начинал с архитектурной визуализации, разрабатывал интерьеры. Однако с детства любил играть в компьютерные игры и стал интересоваться тем, как их создают.

    6 лет назад обучение GameDev вообще не было развито, информацию искал сам: посещал всевозможные курсы, изучал все, что было в Интернете. Делал много тестовых заданий в разные компании, в одну из которых меня и приняли. Во время их выполнения я смог собрать своё портфолио и узнать о требованиях, которые нужны при разработке объектов для игр. Так начался мой путь. Поначалу я занимался тем, что окружает персонажа в игре, а сейчас уже работаю над разработкой уровней для игр.

    Когда ты начинаешь знакомиться с 3D-графикой, то понимаешь, насколько она обширна, насколько много специализаций. Здесь себя может обрести каждый, ведь работы очень много.

    Поступить в университет Польши

    Гарантия поступления в рейтинговые университеты
    Поддержка студентов в Польше

    Узнать больше

    Насколько важно образование по 3D-графике?

    Если 10 лет назад этому не учили в университетах, и диплом не был важным — сейчас все изменилось. В Европе многие вузы имеют это направление, постепенно оно появляется и в Украине. Я бы выделил несколько моментов важности образования по 3D-графике.

    Структурированные знания. Университет выступает более как условная сторона, которая дает фундаментальную подготовку, но параллельно – нужно самому дополнительно учиться, посещать курсы, делать первые разработки. Есть классный сайт Artstation, на котором художники разных стилей выставляют свои работы. Это и определенная социальная сеть, помогающая компаниям находить специалистов со всего мира. Но помните, заказчики из другого уголка планеты не знают украинский, поэтому английский стоит учить.

    Работа в международных компаниях. Сейчас, если программист хочет переехать в США или Германию на работу, образование является важнейшим фактором. К тому же профильное образование. Если человек мега-крутой специалист, как исключение, могут простить отсутствие диплома, но это бывает редко. Приоритеты сейчас на дипломированных специалистах.

    Также, университет – это знакомства, рекомендации и практика в компаниях. В Польше, например, есть известная компания CD Projekt RED — компания разработчик компьютерных игр, выпустившая легендарные: “The Witcher”, “Cyberpunk 20772. Еще во время обучения нужно ставить себе цель попасть туда на работу. Что касается Украины, здесь тоже множество крутых работодателей и здоровая конкуренция на рынке труда. Разница между европейскими и украинскими компаниями в 3D-графике уже не сильно ощутима.

    Дисциплина. Самое главное, что дает университет – это дисциплина. Быстро учиться, овладевать материалом, структурировано изучать все — база, которую предоставит ВУЗы. Главное не думать, что вуз дает все и больше не нужно учиться. Это ошибочное видение.

    Нужно ли уметь рисовать и иметь талант?

    Не каждый художник, работающий в компьютерной графике, умеет классно рисовать от руки. Это умение нужно для 2Д-графики, для концепт-художника. Для специалиста по 3D-графике необходимы навыки понимания пропорций, материала, креативность. Скажу больше, рисование – не закон успеха. Также стоит еще обладать техническими знаниями. К примеру, есть комната, ты творчески в ней расставляешь все предметы, делаешь атмосферу, но есть и техническая сторона: правильное расположение стен, длины и высоты дверей. Поэтому сочетание творческого видения и технических знаний – лучшее для качественного результата.

    Часто в ИТ или 3D-сферу идут ради больших зарплат. В начале, когда ты джуниор, заработные платы невысокие, и чтобы они росли – стоит самому развиваться и много работать. Если ожидаешь просто финансовых наград – результата не будет. Надо целиться на развитие, на желание делать крутой продукт – тогда тебя заметят и начнется карьерный (и финансовый) рост.

    Каких специалистов не хватает: 2D или 3D?

    3D-художники и 2D-художники – необходимы компаниям одинаково. Так как 2D-художник (именно в GameDev) – это человек, рисующий концепции. Например: в одной из программ (используемой при создании 2D-графики (Photoshop) создают персонажа, рисуют его — и это 2D-изображение. Но затем его передают к художнику персонажей, и по этому концепту создадут 3D-модель. Они взаимосвязанные, и потому одинаково популярные на рынке.

    Какие направления в 3D-графике сейчас популярны?

    Game Development – ​​разработка компьютерных игр. Здесь необходимы специалисты всех направлений, даже тестировщики.

    Также, сейчас активно развивается сфера VR (Виртуальной реальности) и игры в VR приобретают большую популярность.

    Какая сейчас ситуация на рынке 3D-графики? Какова динамика развития?

    Такой скорости развития информации, как в ИТ технологиях — нет ни в одной сфере. Это касается и 3D-графики. Ты работаешь в коллективе, где все ежедневно развиваются, проходят обучение, и если ты не успеваешь – то знания просто устареют. Это не та профессия, где выучился, пошел работать – и этого достаточно.

    Факт: 35% того, что я использовал в работе 6 лет назад – уже не актуально.

    На данный момент рынок не перегрет. Очень круто искать те ниши, на которых высок спрос, но низкая популярность среди исполнителей. Например, заходишь на Artstation и видишь, сколько художников создают обычные объекты окружения, такие как – шкаф, дом, авто и другие. А если искать художника по растениям – таких специалистов очень мало. Так что можно выбрать не слишком популярную нишу и чувствовать себя в ней комфортно.

    Как выбрать качественные курсы для дополнительной учебы?

    Когда поступаешь в университет – не всегда знаешь, что именно там будешь изучать. Поэтому нужно заранее себе определить то, какие направления наиболее интересны, в чем хочется развиваться. Ведь после прохождения специализированных курсов будет и сертификат, и знания которые можно добавлять в резюме. Сейчас часто есть реклама курсов, по типу «Изучи за 30 дней программу и работай!». Нет, никто за 30 дней не сделает из вас полноценного специалиста, разве что познакомит с основами работы и программами и все. Учитывайте, кто ведет обучение и как его рекламируют.

    В общем можно выделить 3 критерия выбора курсов:

    • Рекомендации преподавателей, друзей или специалистов;
    • Курсы от компаний. На них дают возможность трудоустройства по окончании. Здесь следует хорошо учиться и заявить о себе, чтобы тебя заметили и взяли на работу;
    • Учиться у специалиста, за которым долго следишь, например, на Artstation или другой платформе.

    Лучше ли работать в узкой специализации или быть многопрофильным специалистом?

    Больше склоняюсь к тому, чтобы быть узкоспециализированным. Когда начинаешь работать по узкой специализации, становишься в ней профессионалом – потом уже можно пополнять багаж знаний и по другим дисциплинам. Таким образом расширяется объем наших знаний. Если же изучать всего понемногу сразу – специалистом стать не получится.

    К тому же очень просто менять специализации, ведь сферы связаны между собой. Принцип разработки оружия и объекта окружения схож: использование техники, иногда и одинакового софта, работа с геометрией. К примеру, работаешь художником по уровням, но захотел пойти в Light Art – обеспечивать освещение в игре. Таких случаев много, и это нормально. Обычно люди не увольняются из компании, а в ней же идут на новую должность и руководство это только поддерживает. Безусловный плюс, что тебе не нужно учиться снова 3 года и всегда можно вернуться к предыдущей работе.

    Советы для тех, кто планирует учить 3D-графику

    Сначала рекомендую посвятить себя обучению в университете, «проникнуться» основами, которые дает вуз. И при изучении – выбирать сферы, в которых есть желание работать.

    Если вы уже изучаете графику в университете, то начинайте создавать свое портфолио. Заходите на сайт компании, где хотите работать, находите вакансии, читаете требования и навыки, которые необходимо иметь и начинаете развивать их. Понимать, что именно нужно учить – это лучший вариант для будущего трудоустройства. На сайтах корпораций всегда есть информация, какие кандидаты им необходимы.

    Определение 3D-графики | ПКМаг

    Создание, отображение и манипулирование объектами на компьютере в трех измерениях. Программы 3D CAD и 3D-графики позволяют создавать объекты в масштабе X-Y-Z (ширина, высота, глубина). Как 3D-объекты, их можно вращать и просматривать под любым углом, а также увеличивать или уменьшать масштаб. Они также позволяют автоматически применять освещение на этапе рендеринга. См. графику, графический конвейер, затенение Гуро, затенение Фонга и 2D-графику. См. также 3D-визуализация.

    3D-сцена В 3D-графике объекты создаются на трехмерной сцене, где текущий вид определяется углом камеры и источниками света, как в реальном мире. (Изображение предоставлено Intergraph Computer Systems.)

    Реклама

    Истории PCMag, которые вам понравятся

    {X-html заменен}

    Выбор редакции

    ЭТО ОПРЕДЕЛЕНИЕ ПРЕДНАЗНАЧЕНО ТОЛЬКО ДЛЯ ЛИЧНОГО ИСПОЛЬЗОВАНИЯ. Любое другое воспроизведение требует разрешения.
    Авторские права © 1981-2023. Компьютерный язык (открывается в новом окне) Co Inc. Все права защищены.

    Информационные бюллетени PCMag

    Информационные бюллетени PCMag

    Наши лучшие истории в вашем почтовом ящике

    Подпишитесь на PCMag

    • Фейсбук (Открывается в новом окне)
    • Твиттер (Откроется в новом окне)
    • Флипборд (Открывается в новом окне)
    • Гугл (откроется в новом окне)
    • Инстаграм (откроется в новом окне)
    • Pinterest (Открывается в новом окне)

    PCMag. com является ведущим авторитетом в области технологий, предоставляющим независимые лабораторные обзоры новейших продуктов и услуг. Наш экспертный отраслевой анализ и практические решения помогут вам принимать более обоснованные решения о покупке и получать больше от технологий.

    Как мы тестируем Редакционные принципы

    • (Открывается в новом окне) Логотип Зиффмедиа
    • (Открывается в новом окне) Логотип Аскмен
    • (Открывается в новом окне) Логотип Экстримтек
    • (Открывается в новом окне) Логотип ИНГ
    • (Открывается в новом окне) Логотип Mashable
    • (Открывается в новом окне) Предлагает логотип
    • (Открывается в новом окне) Логотип RetailMeNot
    • (Открывается в новом окне) Логотип Speedtest
    • (Открывается в новом окне) Логотип Спайсворкс

    (Открывается в новом окне)

    PCMag поддерживает Group Black и ее миссию по увеличению разнообразия голосов в СМИ и прав собственности на СМИ.

    © 1996-2023 Ziff Davis, LLC., компания Ziff Davis. Все права защищены.

    PCMag, PCMag.com и PC Magazine входят в число зарегистрированных на федеральном уровне товарных знаков Ziff Davis и не могут использоваться третьими лицами без явного разрешения. Отображение сторонних товарных знаков и торговых наименований на этом сайте не обязательно указывает на какую-либо принадлежность или поддержку PCMag. Если вы нажмете на партнерскую ссылку и купите продукт или услугу, этот продавец может заплатить нам комиссию.

    • О Ziff Davis(Открывается в новом окне)
    • Политика конфиденциальности(Открывается в новом окне)
    • Условия использования(Открывается в новом окне)
    • Реклама(Открывается в новом окне)
    • Специальные возможности(Открывается в новом окне)
    • Не продавать мою личную информацию (откроется в новом окне)
    • (Открывается в новом окне) доверительный логотип
    • (Открывается в новом окне)

    Компьютерная графика 3D-графика — javatpoint

    следующий → ← предыдущая

    Трехмерные преобразования являются расширениями двумерного преобразования. В 2D используются две координаты, то есть x и y, тогда как в 3D используются три координаты x, y и z.

    Для трехмерных изображений и объектов необходимы трехмерные преобразования. Это переводы, масштабирование и вращение. Они также называются базовыми преобразованиями, представленными с помощью матрицы. Более сложные преобразования выполняются с использованием матрицы в 3D.

    2D может отображать двумерные объекты. Например, линейчатая диаграмма, круговая диаграмма, графики. Но некоторые природные объекты можно представить с помощью 3D. Используя 3D, мы можем видеть разные формы объекта в разных сечениях.

    В 3D, когда выполняется перемещение, нам также нужны три фактора для вращения, это компонент трех вращений. Каждая из них может быть выполнена вдоль любых трех декартовых осей. В 3D мы также можем представить последовательность преобразований в виде одной матрицы.

    Компьютерная графика использует САПР. CAD позволяет манипулировать компонентами машин, которые являются трехмерными. Он также обеспечивает изучение кузовов автомобилей, деталей самолетов. Все эти действия требуют реализма. Для реалистичности требуется 3D. В производстве реалистичной 3D-сцены из 2D сложно. Для этого требуется трехмерность, то есть глубина.

    3D-геометрия

    Трехмерная система имеет три оси x, y, z. Ориентация трехмерной системы координат бывает двух типов. Правосторонняя система и левосторонняя система.

    В правосторонней системе большой палец правой руки указывает на положительное направление z, а в левой системе большой палец указывает на два отрицательных направления. На следующем рисунке показана правосторонняя ориентация куба.

    Использование правой системы координат углов А, В, С, D куба

    Точка A         x, y, z
    Точка B         x, y, 0
    Точка C         0, y, 0
    Точка D         0, y, z

    Создание реализма в 3D: Трехмерные объекты выполнены с использованием компьютерной графики. Техника, используемая для двухмерного отображения трехмерных объектов, называется проекцией. Доступны несколько типов проекции, например,

    1. Параллельная проекция
    2. Перспективная проекция
    3. Орфографическая проекция

    1. Параллельная проекция: В этой проекции точка на экране определяется в пределах точки трехмерного объекта линией, перпендикулярной экрану дисплея. Архитектор Чертеж, т. е. план, вид спереди, вид сбоку, фасад есть не что иное, как линии параллельных проекций.

    2. Перспективная проекция: Эта проекция обладает тем свойством, что дает представление о глубине. Чем дальше объект от зрителя, тем меньше он будет казаться. Все линии в перспективной проекции сходятся в центральной точке, называемой центром проекции.

    3. Орфографическая проекция: Самый простой вид проекции. При этом мы берем вид сверху, снизу и сбоку на объект, извлекая параллельные линии из объекта.

    Трехмерные модели

    Методы создания различных изображений твердого объекта зависят от типа объекта.

    Решение неравенств с двумя модулями: Неравенство с несколькими модулями | Подготовка к ЕГЭ по математике

    Неравенства с модулем

    Продолжаем изучать модуль числа. Сегодня мы научимся решать неравенства с модулем.

    Чтобы решать неравенства с модулем, нужно прежде всего уметь решать простейшие линейные неравенства, а также знать что такое модуль и как его раскрывать.

    Независимо от того, решаем мы уравнение или неравенство, нужно уметь раскрывать модуль.

    Рассмотрим к примеру простейшее неравенство с модулем:

    |x| > 2

    Чтобы решить данное неравенство раскроем его модуль.

    Если подмодульное выражение больше или равно нулю, то исходное неравенство примет вид:

    > 2

    Решением этого неравенства является множество всех чисел, бóльших 2. Отметим их на координатной прямой:

    А если подмодульное выражение меньше нуля, то исходное неравенство примет вид:

    x > 2

    Умнóжим обе части этого неравенства на −1. Тогда полýчим неравенство < −2. Решением этого неравенства является множество всех чисел, мéньших −2. Отметим эти решения на том же рисунке, где мы отметили решения для неравенства > 2

    Забавно, но получившиеся промежутки < −2 и > 2 являются ответом к нашей задаче. Если в исходное неравенство |x| > 2 подставить какое-нибудь значение x, удовлетворяющее данному неравенству, то это значение будет принадлежать промежутку (−∞ ; −2) или промежутку (2 ; +∞).

    То есть решением исходного неравенства является совокупность из < −2 и > 2

    Совокупностью неравенств мы будем называть несколько неравенств, объединённых квадратной скобкой, и которые имеют множество решений, удовлетворяющих хотя бы одному из неравенств, входящих в данную совокупность.

    Чтобы записать окончательный ответ, промежутки < −2 и > 2 следует объединить. В математике знаком объединения служит ∪. Тогда:

    x ∈ (−∞ ; −2) ∪ (2 ; +∞)

    Знак объединения ∪ читается как «или». Тогда запись x ∈ (−∞ ; −2) ∪ (2 ; +∞) можно прочитать так:

    Значение переменной x принадлежит промежутку (−∞ ; −2) или промежутку (2 ; +∞)

    Действительно, если подставить какое-нибудь значение x, являющееся решением исходного неравенства, то это значение будет принадлежать промежутку (−∞ ; −2) или промежутку (2 ; +∞).

    Например, число 3, является решением исходного неравенства |x| > 2

    |3| > 2      3 > 2

    Значение 3 принадлежит промежутку (2 ; +∞). Также оно удовлетворяет хотя бы одному из неравенств совокупности , а именно неравенству x>2.

    Значение −4 тоже является решением исходного неравенства |x| > 2. Это значение принадлежит промежутку (−∞ ; −2)

    |−4| > 2        4 > 2

    Также значение −4 удовлетворяет хотя бы одному из неравенств совокупности , а именно неравенству < −2.

    Согласно определению, модуль числа x есть расстояние от начала координат до точки x. В неравенстве |x| > 2 это расстояние больше чем 2.

    Действительно, от начала координат (точка 0) любое расстояние бóльшее двух, будет решением неравенства |x| > 2

    Ответ: x ∈ (−∞ ; −2) ∪ (2 ; +∞)

    Обратите внимание, что границы −2 и 2 не включены в соответствующие промежутки. Это потому, что при подстановке этих чисел в исходное неравенство, получается неверное неравенство.

    Теперь немного поменяем наш пример. В неравенстве|x| > 2 поменяем знак > на знак <

    |x| < 2

    Решим это неравенство.

    Как и раньше для начала раскрываем модуль. Если подмодульное выражение больше или равно нулю, то получим неравенство < 2. Решениями этого неравенства являются все числа, мéньшие двух. Отметим их:

    А если подмодульное выражение меньше нуля, то получим неравенство −< 2. Умнóжим обе части этого неравенства на −1. Тогда получим неравенство > −2. Решениями этого неравенства являются все числа, бóльшие −2. Отметим эти решения на том же рисунке, где мы отметили решения для неравенства x < 2.

    Для наглядности, решения неравенства > −2 отметим красным цветом:

    Если выражение |x| это расстояние от начала координат до точки x, то неравенство |x| < 2 говорит, что это расстояние меньше чем 2. На рисунке видно, что от начала координат расстояния, мéньшие двух, лежат в промежутках от −2 до 0 и от 0 до 2

    А эти расстояния одновременно будут принадлежать промежуткам < 2 и > −2

    Обратите внимание, что в этот раз промежутки обрамлены знáком системы, а не знáком совокупности как в прошлом примере. Это означает, что значения x одновременно удовлетворяют обоим неравенствам (промежуткам < 2 и > −2)

    То есть решением неравенства |x| < 2 является пересечение промежутков < 2 и > −2. Напомним, что пересечением двух промежутков является промежуток, состоящий из чисел, которые принадлежат как первому промежутку так и второму:

    x ∈  (−2 ; 0) ∩ (0 ; 2)

    Знак пересечения ∩ читается как «и». Тогда запись ∈ (−∞ ; 2) ∩ (−2 ; +∞) можно прочитать так:

    Значение переменной x одновременно принадлежит промежутку (−∞ ; 2) и промежутку (−2 ; +∞)

    Действительно, если подставить какое-нибудь значение x, являющееся решением неравенства |x| < 2, то это значение будет принадлежать одновременно промежутку (−∞ ; 2) и (−2 ; +∞).

    Например, число 1 является решением исходного неравенства |x| < 2

    |1| < 2      1 < 2

    Значение 1 одновременно принадлежит промежутку  (−∞ ; 2) и промежутку (−2 ; +∞)

    Также, значение 1 удовлетворяет обоим неравенствам системы

    А если к примеру подставить значение, не являющееся решением неравенства |x| < 2, то это значение не будет одновременно принадлежать промежуткам (−∞ ; 2) и (−2 ; +∞). Например, значение 7

    |7| < 2      7 < 2

    Несмотря на то, что значение 7 принадлежит одному из промежутков, а именно промежутку (−2 ; +∞), данное значение не является решением исходного неравенства, поскольку оно не удовлетворяет ему. Также, данное значение не принадлежит одновременно обоим промежуткам: (−∞ ; 2) и (−2 ; +∞).

    Для неравенства |x| < 2 ответ можно записать покороче:

    x ∈ (−2 ; 2)

    Из рассмотренных примеров видно, что решением неравенства с модулем может быть либо объединение промежутков либо их пересечение.

    В первом примере мы решили неравенство |x| > 2, то есть неравенство вида |x| > a. Это неравенство при котором модуль больше какого-нибудь числа или буквенного выражения. Решением такого неравенства является объединение решений неравенств, получающихся после раскрытия модуля исходного неравенства. Неравенства, получающиеся после раскрытия модуля, следует записывать в виде совокупности:

    Совокупность свóдится потому, что итоговые решения будут удовлетворять хотя бы одному из неравенств, полученных после раскрытия модуля исходного неравенства.

    Во втором примере мы решили неравенство |x| < 2, то есть неравенство вида |x| < a. От предыдущего неравенства оно отличается только знáком. Но это неравенство при котором модуль меньше какого-нибудь числа или буквенного выражения. Решением такого неравенства является пересечение решений неравенств, получающихся после раскрытия модуля исходного неравенства. Неравенства, получающиеся после раскрытия модуля, следует записывать в виде системы:

    Система записывается потому, что итоговые решения будут удовлетворять обоим неравенствам, полученным после раскрытия модуля исходного неравенства.

    Эти же правила сохраняются и для неравенств, содержащих знаки ≥ и ≤

    Например, решим неравенство |x| ≥ 1. Модуль больше или равен числу. Поэтому решением будет объединение решений неравенств, которые получатся после раскрытия модуля. После раскрытия модуля и выполнения необходимых тождественных преобразований, получим совокупность неравенств  1 и  −1

    Решением служит объединение промежутков  −1 и  1

    x ∈ (−∞ ; −1] ∪ [1 ; +∞)

    Обратите внимание, что границы −1 и 1 включены в соответствующие промежутки. Это потому что при подстановке этих чисел в исходное неравенство, получается верное неравенство.

    Решим теперь к примеру неравенство |x|  1. Модуль меньше или равен числу. Поэтому решением будет пересечение решений неравенств, которые получатся после раскрытия модуля. После раскрытия модуля и выполнения необходимых тождественных преобразований, получим систему неравенства:  1 и  −1

    Решением служит пересечение промежутков  1 и  −1

    x ∈ (−∞ ; 1] ∩ [−1 ; +∞)

    или покороче:

    x ∈ [−1 ; 1]

    Обратите внимание, что границы −1 и 1 включены в соответствующие промежутки. Это потому что при подстановке этих чисел в исходное неравенство, получается верное неравенство.

    Аналогично решаются неравенства, в левой части которого модуль, а справа не просто число, а буквенное выражение.

    Пример 4. Решить неравенство |7 6| < + 12

    Решение

    Для начала раскроем модуль. Вспоминаем, что если неравенство содержит знак < или ≤, то неравенства получившиеся после раскрытия модуля, следует записать в виде системы. Это будет означать, что итоговые решения будут удовлетворять обоим неравенствам.

    Итак, после раскрытия модуля получим следующую систему:

    В данном случае система содержит не совсем элементарные неравенства как в прошлых примерах. Данные неравенства следует упростить, используя известные тождественные преобразования.

    Раскроем скобки во втором неравенстве. Тогда получим следующую систему:

    В обоих неравенствах выражения, содержащие неизвестные, перенесём в левую часть, а числовые выражения — в правую. Затем приведём подобные слагаемые. Тогда получим систему:

    В первом неравенстве разделим обе части на 6. Во втором неравенстве разделим обе части на −8. Тогда получим окончательную систему:

    Изобразим решения на координатной прямой:

    Решением является пересечение промежутков (−∞ ; 3) и , то есть промежуток

    Ответ:


    Пример 5. Решить неравенство |1 − 2x| ≥ 4 − 5x

    Решение

    Для начала раскроем модуль. Вспоминаем, что если неравенство содержит знак > или ≥, то неравенства получившиеся после раскрытия модуля, следует записать в виде совокупности:

    После раскрытия модуля получим следующую совокупность:

    Выполним необходимые тождественные преобразования в обоих неравенствах. В результате получим:

    Изобразим решения на координатной прямой:

    Решением является объединение промежутков и [1 ; +∞), то есть промежуток

    Ответ: .

    Задание 1. Решить неравенство:

    Решение:

    Ответ: x ∈ (−36 ; 36).

    Показать решение

    Задание 2. Решить неравенство:

    Решение:

    Ответ: x ∈ (−∞ ; −2) ∪ (2 ; +∞).

    Показать решение

    Задание 3. Решить неравенство:

    Решение:

    Ответ:

    Показать решение

    Задание 4. Решить неравенство:

    Решение:

    Ответ: x ∈ [−5 ; 2]

    Показать решение

    Задание 5. Решить неравенство:

    Решение:

    Ответ: x ∈ (−∞ ; 0)

    Показать решение

    Задание 6. Решить неравенство:

    Решение:

    Ответ:

    Показать решение


    Понравился урок?
    Вступай в нашу новую группу Вконтакте и начни получать уведомления о новых уроках

    Возникло желание поддержать проект?
    Используй кнопку ниже

    Опубликовано

    25.

    Неравенства с модулем

    I тип: Неравенство содержит некоторое выражение под модулем и число вне модуля:

    где (3.27)

    Решение зависит от знака числа А.

    1. Если то неравенство (3.27) не имеет решений.

    2. Если то неравенство (3.27) равносильно системе неравенств

    где (3.28)

    1. Если то неравенство (3.28) не имеет решений.

    2. Если то неравенство (3.28) равносильно уравнению

    3. Если , то неравенство (3.28) равносильно системе неравенств

    где (3.29)

    1. Если то решением неравенства (3.29) является множество всех значений Х из ОДЗ выражения

    2. Если то решением неравенства (3.29) является множество всех значений Х из ОДЗ выражения таких, что

    3. Если то неравенство (3.29) равносильно совокупности

    где (3.30)

    1. Если то решением неравенства (3.30) является множество всех значений Х из ОДЗ выражения

    2. Если то неравенство (3. 30) равносильно совокупности

    II тип: Неравенство, которое содержит выражение с переменной под знаком модуля и вне его:

    (3.31)

    Где – некоторые выражения с переменной Х.

    Для решения неравенств типа (3.31) можно использовать следующие способы.

    1-й способ: используя определение модуля, получаем равносильную совокупность систем:

    2-й способ: Решаем аналогично решению неравенства (3.29) при дополнительном ограничении на знак выражения

    1. Если

    (3.32)

    То решением является множество всех значений Х из ОДЗ выражения которые удовлетворяют условию (3.32).

    2. Если

    То решением является множество всех значений Х, которые удовлетворяют системе

    3. Если решение определяется системой

    Ответом в решении неравенства (3.31) является объединение всех решений, полученных на этапах 1–3.

    3-й способ: метод интервалов.

    Для решения необходимо:

    1) найти значения Х, для которых

    2) найденные значения Х нанести на числовую ось;

    3) определить знак выражения на всех полученных промежутках;

    4) нарисовать кривую знаков;

    5) раскрыть модуль, пользуясь рисунком, и получить соответствующее неравенство, которое следует решить вместе с условием принадлежности переменной Х определенному промежутку;

    6) в ответе неравенства указать совокупность полученных решений.

    III тип: Неравенство содержит несколько модулей и решается двумя способами:

    1-й способ: Можно использовать определение модуля и решать совокупность систем неравенств. Этот способ, как правило, не является рациональным.

    2-й способ: использовать метод интервалов. Необходимо нарисовать столько числовых осей и кривых знаков, сколько модулей содержится в неравенстве. Для каждого промежутка следует решать полученное после раскрытия модулей неравенство при условии, что переменная Х принадлежит конкретному промежутку. В ответе указывают объединение всех полученных решений.

    IV тип: Неравенство вида

    где (3.33)

    Решается двумя способами:

    1-й способ: метод интервалов.

    2-й способ: согласно теореме равносильности (см. свойства равносильности неравенств (3.22) и (3.23)) неравенство (3.33) можно возводить в квадрат:

    Решение неравенства (3.33) сводится к решению неравенства

    Аналогично решают неравенства IV типа (3. 33), если они заданы со знаками

    V тип: Неравенства, решаемые заменой переменной.

    В таком случае выражение с модулем обозначают новой переменной. Неравенство с новой переменной решают до конца (т. е. до возможного получения промежутков решения для новой переменной). Затем возвращаются к старой переменной и решают полученные неравенства с модулем как неравенства I типа.

    Пример 1. Решить неравенства:

    1) 2)

    3) 4)

    5) 6)

    Решение. 1) Решаем как неравенство I типа:

    Получаем ответ:

    2) Решаем как неравенство I типа:

    Второе неравенство совокупности не имеет решения (соответствующая парабола лежит над осью Ох). Первое неравенство сводится к виду

    Его решение: это и есть ответ.

    3) Решаем как неравенство II типа. Оно имеет решение, если Поэтому получаем равносильную систему:

    Получаем ответ:

    4) Заданное неравенство может быть записано в виде

    Заменим переменную Решаем неравенство

    Его решение

    Возвращаемся к переменной Х и решаем совокупность

    Получаем

    Т.  е. приходим к ответу

    5) Для решения неравенства используем метод интервалов. Запишем неравенство в виде

    Построим числовые прямые и определим знаки выражений, стоящих под модулем (рис. 3.10).

    ОДЗ:

    Рис. 3.10

    А) рассмотрим неравенство на 1-м промежутке. Получаем систему

    (3.34)

    Решаем неравенство

    Получаем

    Система (3.34) сводится к системе

    На данном промежутке решений нет.

    Б)

    Если , то С учетом рассматриваемого промежутка имеем:

    Получаем

    В)

    Решением является промежуток:

    Объединим полученные решения и приходим к ответу:

    6)

    ОДЗ:

    Введем новую переменную:

    тогда и приходим к неравенству вида

    Решаем его

    Используем метод интервалов (рис. 3.11).

    Рис. 3.11

    Запишем полученное решение в виде совокупности:

    Вернемся к переменной Х:

    (3.35)

    – выполняется при любых

    С учетом ОДЗ второе неравенство системы (3.35) равносильно системе

    Получаем ответ:

    < Предыдущая   Следующая >

    абсолютное значение — Двойной модуль в неравенстве

    спросил

    Изменено 10 лет, 6 месяцев назад

    Просмотрено 2к раз

    $\begingroup$

    Может кто-нибудь объяснить мне (или дать ссылку на сайт, который делает), как решить эту проблему? $$ ||х+1| -1| \geq 3 $$ Я понятия не имею, как вычислить этот знак двойного абсолютного значения.

    • неравенство
    • абсолютное значение

    $\endgroup$

    1

    $\begingroup$

    Работа снаружи внутрь. Для начала у вас есть неравенство $|u-1|\ge 3$, где $u=|x+1|$. Это эквивалентно $$u-1\le -3\quad\mathbf{or}\quad u-1\ge 3\;.$$ Решая их, мы находим, что $$u\le-2\quad\mathbf {или}\quad u\ge 4\;,$$, что в терминах исходной переменной $x$ равно

    $$|x+1|\le -2\quad\mathbf{or}\quad|x+1|\ge 4\;.\tag{1}$$

    Абсолютное значение невозможно отрицательно, поэтому $(1)$ сводится к $|x+1|\ge 4$. Как и в самом первом шаге, это эквивалентно

    $$x+1\le-4\quad\mathbf{or}\quad x+1\ge 4\;,$$, решение которого равно

    $$x \le-5\quad\mathbf{or}\quad x\ge 3\;.$$

    $\endgroup$

    4

    $\begingroup$

    Так как у тебя уже есть раствор снаружи, я открою тебе изнанку.

    • Сначала возьмем случай $x+1\geq 0$. Таким образом, $||x+1|-1|=|x+1-1|=|x|$.

      $|x|\geq3$ имеет решение $x\leq-3$ или $x\geq3$. Зная, что $x\geq-1$, первое невозможно.

    • Теперь рассмотрим $x+1\leq0$. Таким образом, $||x+1|-1|=|-(x+1)-1|=|-x-2|=|x+2|$. $|x+2|\geq3$ имеет решение $x\leq-5$ или $x\geq1$. Зная, что $x\leq-1$, второе невозможно.

    • 92=(а-б)(а+б)\\ &\Longleftrightarrow (x-3)(x+5)\geq 0\\ &\Longleftrightarrow x\in(-\infty,-5]\cup[3,+\infty) \end{выравнивание} $$

      $\endgroup$

      $\begingroup$

      Если бы я был перед классом, который столкнулся с этой проблемой, я бы изобразил ее на графике, проанализировав ее изнутри. Во-первых, $|x+1|$ имеет V-образный граф с вершиной в $(-1,0)$. Тогда $|x+1|-1$ также имеет V-образный граф с вершиной в $(-1,-1)$. Обратите внимание, что этот график пересекает ось $x$ в точке $(-2,0)$ и начале координат. Когда вы берете абсолютное значение функции $|x+1|-1$, часть графика, расположенная ниже оси $x$, отражается выше. Результат теперь W -образный график с левым углом в $(-2,0)$, средним углом в $(-1,1)$ и прямым углом в начале координат. Слева от $(-2,0)$ график поднимается (по мере удаления от начала координат) с наклоном $-1$, т.е. имеет уравнение $y=-x-2$, а справа от начала координат, график поднимается с наклоном $1$, т.е. просто $y=x$. Чтобы значение этой функции было не меньше $3$, нужно справа $x\ge3$, а слева $x\le-5$.

      $\endgroup$

      Как решать неравенства с модулем

      Как решать неравенства с модулем :

      В этом разделе мы узнаем, как решить неравенство с модулем.

      Решение неравенств по модулю — концепция

      Если задан вопрос в любой из следующих форм, мы должны следовать данным методам для решения x.

      Вопросы в форме

      Первый шаг, который нужно сделать

      Решение

       |х — а| <  r

      -г < х - а < г

      (-р + а, г + а)

      |х — а| ≤ г

      -r ≤  x — a ≤  r

      [а- г, а + г]

      |х — а| > r

      х — а < -r

       и 

      х — а > r

      (∞,a-r)U(a+r,∞)

      |х — а| ≥  р

      х — а ≤ -r

      и

      х — а ≥ r

      (∞,а-р]U[а+r,∞)

      Пример 1 < 2

      и выразить решение в виде интервалов.

      Решение:

      -2 < x - 9 < 2

      Добавьте 9 к уравнению

      -2 + 9 < x - 9 + 9 < 2 + 9

      7 < x < 11

      неравенство (7, 11).

      Пример 2 :

      Решите абсолютное неравенство, приведенное ниже

      |2/ (x — 4)| > 1 , x ≠ 4

      и выразить решение в виде интервалов.

      Решение:

      Из данного неравенства получаем, что 2 > (x — 4)

      -2 < x - 4 < 2

      Добавьте 4 по всему неравенству

      -2 + 4 < x - 4 + 4 < 2 + 4

      2 < x < 6

      Мы не можем выразить решение в виде (2 , 6). Потому что в середине 2 и 6 у нас есть значение 4.

      Итак, мы должны разбить его на два интервала.

      (2, 4) U (4, 6)

      Пример 3 :

      Решите абсолютное неравенство, приведенное ниже

      |3 — (3x/4)| ≤  1/4

      и выразить решение в виде интервалов.

      Решение:

      (-1/4) ≤ 3 — (3x/4)  ≤ (1/4)

      (-1/4) ≤ (12 — 3x)/4  ≤ (1/4)

      Умножить на 4 во всем уравнении

      -1 ≤ (12 — 3x) ≤ 1

      Вычесть 12 во всем уравнении

      -1 — 12 ≤ 12 — 3x — 12 ≤ 1 -12

      -13 ≤ — ≤ -11

      Делится на (-3) во всем уравнении

      -13/(-3) ≤ — 3x  ≤ -11

      13/3 ≤ x ≤ 11/3

      11/3 ≤ x ≤ 13/ 3

      Следовательно, набор решений приведенного выше абсолютного неравенства равен [11/3, 13/3].

      Дробное число в двоичной системе: Перевод дробных чисел из десятичной системы счисления в любую другую систему счисления — урок. Информатика, 10 класс.

      {-2}\). Поскольку целая часть у нормализованного представления числа всегда (кроме числа 0) равна 1, то целая часть не хранится, а хранится только дробная часть мантиссы. Количество значащих знаков мантисcы, которое хранится, может быть различным для различных способов представления действительных чисел. Например, в типе данных двойной точности, который в языке Python соответствует типу float, в языке C — типу double, в языке Pascal — типу double, хранится 52 бита дробной части мантисы.

      Стандарт IEEE754 определяет несколько типов представления действительных чисел, основными из которых являются числа одинарной точности (для их хранения требуется 4 байта), двойной точности (8 байт) и расширенной точности (10 байт).

      Точность

      Размер (байт)

      C, C++

      Pascal

      Python

      Мантисса (бит)

      Точность,
      десятичных цифр

      Максимальное
      значение

      Минимальное
      положительное
      значение

      одинарная

      4

      float

      single

      23

      \(\approx 7{,}2\)

      \(3{,}4\cdot10^{38}\)

      \(1{,}4\cdot10^{-45}\)

      двойная

      8

      double

      double

      float

      52

      \(\approx 15{,}9\)

      \(1{,}7\cdot10^{308}\)

      \(5{,}0\cdot10^{-324}\)

      расширенная

      10

      long double
      (только в gnu gcc/g++)

      extended

      63

      \(\approx 19{,}2\)

      \(1{,}1\cdot10^{4932}\)

      \(1{,}9\cdot10^{-4951}\)

      Неточность при представлении действительных чисел

      Рациональные числа, которые не могут быть представлены в виде дроби со знаменателем, являющимся степенью двойки, не могут быть точно представлены в виде конечной двоичной дроби, а, значит, не могут быть в точности представлены в памяти компьютера. {-2}\) и хранятся только 52 цифры дробной части мантиссы числа. Дальнейшие цифры отбрасываются (с округлением), поэтому вместо числа \(\frac{1}{3}\), которое непредставимо в действительных типах данных, хранится другое ближайшее к нему представимое число. В случае с числом \(\frac{1}{3}\) записывается меньшее число, также меньшее число записывается и при записи в переменную двойной точности значений \(\frac{1}{10}\) и \(\frac{2}{10}\), а вот вместо числа \(\frac{3}{10}\) уже будет записано большее представимое число. Поэтому если записать в две переменные значения \(0.1\) и \(0.2\) (десятичные), а в третью переменную записать их сумму, то результат окажется меньше, чем \(\frac{3}{10}\), в то время как при записи в переменную значения \(\frac{3}{10}\) явно в виде \(0.3\), получится результат, больший чем \(\frac{3}{10}\). То есть при вычислении суммы чисел \(0.1\) и \(0.2\) результат получится отличным от значения \(0.3\), записанного в переменную явно, то есть с точки зрения компьютерной арифметики \(0. 1 + 0.2 \ne 0.3\). Эту проблему (неточное представление действительных чисел и выполнение арифметических операций с действительными числами) всегда следует иметь в виду.

      Использование эпсилон при сравнении действительных чисел

      Таким образом, сравнивать действительные числа в компьютерных программах на точное равенство нельзя. Вместо этого если нужно сравнить два числа \(a\) и \(b\) на равенство, правильным будет проверка условия, что эти два числа не сильно различаются, то есть что модуль их разности не превосходит некоторого небольшого значения \(\varepsilon\), то есть что \(|a-b|<\varepsilon\). Значение \(\varepsilon\) как правило определяется для каждой задачи исходя из той точности, с которой необходимо получить результат.

      В следующей таблице указано, как нужно проверять различные сравнения действительных чисел с использованием \(\varepsilon\).

      Условие

      Как нужно проверять

      \(a=b\)

      \(|a-b|\lt\varepsilon\)

      \(a\ne b\)

      \(|a-b|\ge \varepsilon\)

      \(a\lt b\)

      \(a \le b — \varepsilon\)

      \(a \le b\)

      \(a \lt b + \varepsilon\)

      \(a\gt b\)

      \(a \ge b + \varepsilon\)

      \(a\ge b\)

      \(a \gt b — \varepsilon\)

      Специальные значения действительных типов

      При работе с действительными числами, как правило, в результате алгоритмических ошибок, возникают некоторые специальные значения, например, inf и nan.

      Значение inf (от infinity) возникает при переполнении действительной переменной, когда результат становится очень большим. Например, если взять положительное число и умножать его на два в цикле, то довольно скоро получится значение inf. Максимальное значение, которое может быть сохранено до получения переполнения, различается для разных типов действительных чисел.

      Значение inf отличается тем, что после получения результата inf из него уже нельзя получить “обычное” действительное число. При прибавлении и вычитании к inf любого действительного (неспециального) числа, умножении и делении inf на положительное число, в результате всегда получается inf. То есть inf считается “очень большим”, настолько, что сложение, умножение и деление все равно оставляет результат “очень большим”. Значение inf считается больше любого действительного неспециального числа.

      Также есть отрицательное значение -inf, которое считается “минус бесконечностью”.

      При ряде операций со значением inf может возникнуть другое специальное значение nan (от not a number — не число). Примеры операций, дающих в результате nan:

      1. \(inf — inf\)
      2. \(inf \times 0\)
      3. \(inf / inf\)

      nan означает, что результат может быть “чем угодно”, то есть это непредсказуемая величина и работать с ней невозможно. Любые арифметические операции c nan будут давать в результате nan. Более того, при любом сравнении nan с любым действительным числом при помощи любой из операций ==, <, <=, >, >= в результате будет получаться false. А при сравнении nan с любым действительным числом при помощи операции != всегда будет получаться true. И даже если значения двух переменных a и b оба равны nan, то сравнение a == b будет давать false, а сравнение a != b будет давать true.

      В заголовочном файле cmath определены две константы INFINITY и NAN, которые принимают значения inf и nan соответственно, а также две функции bool isinf(x) и bool isnan(x), которые возвращают true, если аргумент x равен inf и nan соответственно или false в противном случае.

      Электронный справочник по ИНФОРМАТИКЕ (Автор Панов В.А.)


      Двоичная система счисления

       

      Введение

      Перевод чисел

      Дробные числа

      Отрицательные числа

      Двоичная арифметика

       

       

       

       

       

       

       

       

       

       

      Двоичная система счисления является основной системой представления информации в памяти компьютера.

      В этой системе счисления используются две цифры: 0 и 1.

      Двоичную цифру называют битом.

      Первое опубликованное обсуждение двоичной системы счисления принадлежит испанскому священнику Хуану Карамюэлю Лобковицу ( 1670 г .). Всеобщее внимание к этой системе привлекла статья немецкого математика Готфрида Вильгельма Лейбница, опубликованная в 1703 г . В ней пояснялись двоичные операции сложения, вычитания, умножения и деления. Лейбниц не рекомендовал использовать эту систему для практических вычислений, но подчёркивал её важность для теоретических исследований. Со временем двоичная система счисления становится хорошо известной и получает развитие. Большинство современных электронно-вычислительных машин используют в своей работе именно эту систему чисел.

       

      Ей было тысяча сто лет.
      Она в сто первый класс ходила,
      В портфеле по сто книг носила —
      Всё это правда, а не бред.
      Когда пыля десятком ног,
      Она шагала по дороге,
      За ней всегда бежал щенок
      С одним хвостом, зато стоногий.
      Она ловила каждый звук
      Своими десятью ушами,
      И десять загорелых рук
      Портфель и поводок держали.
      И десять тёмно-синих глаз
      Рассматривали мир привычно…

      Но станет всё совсем обычным,
      Когда поймёте наш рассказ.

      ———————

      С помощью двоичной системы кодирования можно зафиксировать любые данные и знания. Это легко понять, если вспомнить принцип кодирования и передачи информации с помощью азбуки Морзе.

      Телеграфист, используя только два символа этой азбуки — точки и тире, может передать практически любой текст.

      Объём памяти компьютера измеряется в байтах. Каждый байт может выражать букву, число, пробел, знак препинания или какой-либо другой символ. Количество символов, которые компьютер может хранить в оперативной памяти, меняется в широких пределах от вида компьютера и его модели.

      Объём памяти, хотя он и измеряется в байтах, обычно выражается в килобайтах. Слово «килобайт», вообще говоря, означает «1000 байт». (Напомним, что приставка «кило» означает «тысяча».)

      Фактически же килобайт равен 1024 байтам: 1 Кбайт = 1024 байт.

      Компьютер с объёмом памяти в 64 К может хранить 64 х 1024 = 65536 символов.

      Объём памяти первых микрокомпьютеров составлял всего лишь 2 Кб. Нынешние компьютеры имеют объём памяти 128, 256, 512, 1024 Мб и более

      Объём памяти новейших компьютеров так велик, что она выражается в гигабайтах, т. е. в миллиардах байтов.

      1 Мбайт = 1024 Кбайт = 1 048 576 байт.

      Итак, каждый символ алфавитно-цифровой информации представляется в компьютере кодом из восьми двоичных цифр. Следовательно, каждый символ в компьютере имеет код объёмом 1 байт.

      имеет в двоичной форме объём 25 байт: 23 буквы и 2 символа «пробел» по 1 байту.

      Пример. Измерим в байтах объём текстовой информации в книге из 258 страниц, если на одной странице размещается в среднем 45 строк по 60 символов (включая пробелы). Один символ в двоичной форме содержит 1 байт. Строка будет содержать 61 байт, учитывая и служебный символ окончания строки. Тогда

      61 байт * 45 строк = 2745 байт.

      Так как в книге 258 страниц текста и на каждой странице в среднем по 2745 байт информации, то объём алфавитно-цифровой информации в книге

      2745 байт * 258 страниц = 708210 байт » 692 Кбайт

      Таким образом, текст книги имеет объём около 692 Кбайт.

       

      Перевод чисел

      Для перевода десятичного числа в двоичное надо разделить его на 2 и собрать остатки, начиная с последнего частного.

      7310 = 10010012

      А вот как происходит перевод двоичного числа в десятичное:

       

       

      В любой системе счисления нужно уметь представлять не только целые числа, но и дробные. С математической точки зрения это ординарная задача, которая давно решена. Однако с точки зрения компьютерной техники это далеко не тривиальная проблема, во многом связанная с архитектурой компьютера. Ресурсы компьютеров не бесконечны, и основной трудностью является представление периодических и непериодических дробей. Следовательно, такие дроби следует округлять, задавать класс точности участвующих (и могущих появиться в результате вычислений!) чисел без потери точности вычислений, а также следить за тем, чтобы потеря точности не произошла при переводе чисел из одной системы счисления в другую. Особенно важно аккуратно производить вычисления при операциях с плавающей точкой.

      Запишем формулу представления дробного числа в позиционной системе счисления:

       

      Ap = an-1·pn-1+an-2·pn-2 + … + a1·p1+a0·p0 +a-1·p-1+a-2·p-2 + . .. + a-m·p-m,       

      В случае десятичной системы счисления получим:

      24,7310 = (2·101+4·100+7·10-1+3·10-2)10

      Перевод дробного числа из двоичной системы счисления в десятичную производится по следующей схеме:

      101101,1012 = (1·25+0·24+1·23+1·22+0·21+1·20+1·2-1+0·2-2+1·2-3)10=45,62510

      Перевод дробного числа из десятичной системы счисления в двоичную осуществляется по следующему алгоритму:

      ·  Вначале переводится целая часть десятичной дроби в двоичную систему счисления;

      ·  Затем дробная часть десятичной дроби умножается на основание двоичной системы счисления;

      ·  В полученном произведении выделяется целая часть, которая принимается в качестве значения первого после запятой разряда числа в двоичной системе счисления;

      ·  Алгоритм завершается, если дробная часть полученного произведения равна нулю или если достигнута требуемая точность вычислений. В противном случае вычисления продолжаются с предыдущего шага.

      Пример: Требуется перевести дробное десятичное число 206,116 в дробное двоичное число.

      Перевод целой части дает 20610=110011102 по ранее описанным алгоритмам; дробную часть умножаем на основание 2, занося целые части произведения в разряды после запятой искомого дробного двоичного числа:

       

      .116 • 2 = 0.232

      .232 • 2 = 0.464

      .464 • 2 = 0.928

      .928 • 2 = 1.856

      .856 • 2 = 1.612

      . 612 • 2 = 1.224

      .224 • 2 = 0.448

      .448 • 2 = 0.896

      .896 • 2 =1.792

      .792 • 2 = 1.584

      и т.д.

      Получим: 206,11610=11001110,00011100112

      Таблицу степеней первых восьми отрицательных степеней двойки

       

      Степень основания

      2

      8

      16

      0

      1

      1

      1

      1

      2

      8

      16

      2

      4

      64

      256

      3

      8

      512

      4096

      4

      16

      4096

      65536

      5

      32

      32768

      1048576

      6

      64

      262144

      16777216

      7

      128

      2097152

      268435456

      8

      256

      16777216

      4294967296

      9

      512

      134217728

      68719476736

      10

      1024

      1073741824

      1099511627776

      11

      2048

      8589934552

      17592186044416

      12

      4096

      68719476736

      281474976710656

      13

      8192

      549755813888

      4503599627370496

      14

      16384

      4398046511104

      72057594037927936

      15

      32768

      35184372088832

      1152921504606846976

      16

      65536

      281474976710756

      18446744073709551616

       

      Отрицательные числа

      Перейдем теперь к вопросу представления отрицательных чисел. Для определенности рассмотрим тип byte, в котором любое число занимает ровно восемь бит. Из записи в двоичной системе счисления равенства (- 1) + 1 = 0 легко найти, какой вид должно иметь неизвестное нам пока двоичное представление xxxxxxxx числа — 1:

          xxxxxxxx + 00000001 = 00000000      

      Ясно, что на месте символов xxxxxxxx должно быть расположено число 11111111. Правильным результатом при этом, конечно, следовало бы считать 100000000, а не 00000000, но ведь мы имеем дело с типом byte и, так как результат обязан разместиться в байте, единица <<исчезает>>.

      Итак, число — 1 должно кодироваться как 11111111. Дальнейшее уже совсем просто: для получения — 2 нужно — 1 уменьшить на единицу, что даст 11111110; число — 3 представляется как 11111101 и т. д.

      Отрицательные числа всегда имеют в своем двоичном представлении единицу в самом старшем разряде, который поэтому называют знаковым, а абсолютная величина кодируемого числа получается как двоичное дополнение остальных бит (нули нужно заменить на единицы и наоборот), увеличенное на один.

      Легко видеть, что при этом самым маленьким отрицательным числом, которое принадлежит типу byte, является число — 128 (двоичное представление 10000000), а самым большим — число 127 (представление 01111111). Все представимыe числа (а их 256) в данном случае могут быть получены как пересечение двух множеств: множества Z всех целых чисел и отрезка [ — 128; 127 ].

      Интересным является следующее наблюдение: если число 01111111 увеличить на единицу, то получится 10000000, что означает следующее:

      127 + 1 = — 128 !!!

      Итак, множество элементов типа byte можно представлять себе в виде свернутого в кольцо отрезка
      [ — 128; 127 ].

      То, что для элементов множества , являющегося машинным аналогом Z, нарушено фундаментальное свойство целых чисел X + 1 > X, способно привести к различным невероятным на первый взгляд результатам, однако гораздо более странные вещи происходят при работе с вещественными числами.

       

       

      Двоичная арифметика

      Над числами в двоичной системе счисления можно выполнять арифметические действия.

      При этом используются следующие таблицы:

      Сложение

      Вычитание

      Умножение

      0+0=0

      0-0=0

      0*0=0

      1+0=1

      1-0=1

      1*0=0

      0+1=1

      1-1=0

      0*1=0

      1+1=10

      10-1=1

      1*1=1

      Рассмотрим примеры:

      1 1 0 1 0 12 + 1 1 0 1 12

       

      1 1 0 1 12 — 1 1 0 12

       

      1 1 0 1 12 * 1 0 12

      Для деления в двоичной системе счисления нужно уметь сравнивать числа (определять, какое больше) и хорошо вычитать. Посмотри деление на анимированном примере

      Пример:

       

      Еще несколько примеров:

       

      Несколько примеров для тренировки:

       

      Простая математика, лежащая в основе алгоритмов преобразования десятичных чисел в двоичные

      Если вы поищете в Интернете «Как преобразовать десятичные числа в двоичные», вы найдете четыре простых алгоритма: два для целых чисел и два для дробей. Они представлены на примерах ниже в первой части статьи. Но хотя почти всегда достаточно просто знать алгоритмы, я решил попытаться понять, почему они работают. Во второй части этой статьи объясняется самая основная математика, стоящая за каждым из них. Знание этого может помочь вам вспомнить любой из алгоритмов, если вы вдруг забудете их. Я настоятельно рекомендую вам взять блокнот и ручку и выполнять операции вместе со мной, чтобы лучше запомнить математику. Вот четыре алгоритма с примерами, которые вы можете найти в Интернете.

      Преобразование десятичного целого числа в двоичное

      Чтобы преобразовать целое число в двоичное, начните с рассматриваемого целого числа и разделите его на 2, обращая внимание на частное и остаток. Продолжайте делить частное на 2, пока не получите частное, равное нулю. Затем просто выпишите остатки в обратном порядке.

      Вот пример такого преобразования с использованием целого числа 12.
      Сначала разделим число на два, указав частное и остаток:

      Теперь осталось просто выписать остаток в обратном порядке — 1100 . Итак, 12 в десятичной системе представляется как 1100 в двоичной.

      Преобразование десятичной дроби в двоичную

      Чтобы преобразовать дробь в двоичную, начните с рассматриваемой дроби и умножьте ее на 2 , обращая внимание на полученную целую и дробную части. Продолжайте умножать на 2, пока не получите в результате дробную часть, равную нулю. Затем просто выпишите целые части из результатов каждого умножения.

      Вот пример такого преобразования с использованием дроби 0,375 .

      Теперь просто выпишем полученную целую часть на каждом шаге — 0,011 . Итак, 0,375 в десятичной системе представляется как 0,011 в двоичной.

      Только дроби со знаменателем, равным степени двойки, могут быть конечно представлены в двоичной форме. Например, знаменатели 0,1 (1/10) и 0,2 (1/5) не являются степенями двойки, поэтому эти числа не могут быть конечно представлены в двоичном формате. Чтобы сохранить их как числа с плавающей запятой IEEE-754, их необходимо округлить до количества доступных битов для мантиссы — 10 бит для половинной точности, 23 бита для одинарной точности или 52 бита для двойной точности. В зависимости от того, сколько битов точности доступно, приближения с плавающей запятой 0,1 и 0,2 могут быть немного меньше или больше, чем соответствующие десятичные представления, но никогда не равны. Из-за этого у вас никогда не будет 0,1+0,2 == 0,3.

      Преобразование двоичного целого числа в десятичное

      Чтобы преобразовать двоичное целое число в десятичное, начните слева. Возьмите текущую сумму, умножьте ее на два и добавьте текущую цифру. Продолжайте до тех пор, пока не останется цифр. Вот пример такого преобразования с использованием дроби 1011 .

      Преобразование целой дроби в десятичную

      Чтобы преобразовать двоичную дробь в десятичную, начните справа с суммы 0. Возьмите текущую сумму, добавьте текущую цифру и разделите результат на 2. Продолжайте, пока не останется цифр. . Вот пример такого преобразования с использованием дроби 0,1011 . Я просто заменил деление на 2 умножением на 1/2 .

      Вот вам 4 простых алгоритма, которые позволят вам преобразовывать двоичные числа в десятичные и обратно.

      Расширение числа по основанию q

      Ключом к пониманию того, почему эти алгоритмы работают, является расширение числа по основанию q числа. Целое число в любой системе счисления может быть представлено в следующем виде:

      где

      • N целое число
      • x цифра 82
      • q — базовое значение (10 для системы с основанием 10, 2 для системы с основанием 2 ) , или просто базовое расширение q . Посмотрим, как выглядит число 12 в десятичной и двоичной системах счисления:

        Аналогично дробное число в любой системе счисления может быть представлено в следующем виде:

        где,

        • 4 — дробь

        • x — цифра (от 0 до 9 для системы счисления по основанию 10, 0 и 1 для системы с основанием 2)
        • q — базовое значение (10 для системы счисления по основанию 10, 2 для системы с основанием 2)

        Для номера 2

        14 в десятичной и двоичной системах представления выглядит следующим образом:

        Преобразование десятичного целого числа в двоичное

        Как оказалось, мы можем использовать эту форму расширения base-q для преобразования числа из десятичной системы в двоичную. Давайте сделаем это для того же числа 12 . Во-первых, давайте притворимся, что мы не знаем, как оно представлено в двоичном формате, и запишем его, заменив неизвестные цифры на x :

        Наша задача — найти все x . Давайте посмотрим, что мы можем сделать здесь. Первое, на что мы должны обратить внимание, это то, что все слагаемые, кроме последнего, будут четными числами, потому что все они кратны двум. Теперь, используя эту информацию, мы можем вывести цифру для 9.0080 x0 ​​ если преобразуемое целое четное, то x0 ​​ 900 , если нечетно — то x0 ​​ должен быть 1 . Здесь у нас четное число 12, поэтому x0 ​​ равно нулю. Давайте запишем эту информацию:

        Далее нам нужно найти значение для x1 . Поскольку все слагаемые от x1 до xN кратны двум, мы можем выделить 9014 . выделить x1 . Сделаем так:

        Также легко увидеть, что сумма значений в скобках равна 6 . Итак, мы можем записать наш первый шаг как:

        Продолжим поиск оставшихся x . Мы можем записать многочлен в скобках как отдельное утверждение:

        Здесь, применяя ту же логику, что и выше, мы видим, что x1 равно 0 . Давайте перепишем его и снова вынесем 2:

        Итак, наш второй шаг:

        Теперь мы можем увидеть закономерность. Мы можем продолжить факторинг 2 , пока частное не станет равным нулю. Давайте последуем этой схеме и посмотрим, что у нас получится.

        Поскольку частное равно 1, осталось только одно слагаемое, поэтому давайте перепишем предыдущее выражение:

        Итак, наш третий шаг:

        Итак, мы получаем следующее:

        Понятно, что x3 равно 1 . Но, так как для нашего алгоритма нам нужно частное, перепишем предыдущее выражение так, чтобы оно имело частное:

        Поскольку мы получили частное 0 , больше не с чем работать, и это был наш последний шаг. Запишем:

        Итак, мы закончили преобразование. Вот как выглядит наше преобразование по шагам:

        Теперь понятно, что остаток на каждом шаге соответствует значению x на соответствующих позициях: первый остаток соответствует первому x, второй остаток второму х и так далее. Итак, число 12 в двоичном виде с использованием описанного выше алгоритма представляется как 1100 .

        Помните, что мы начали с идеи показать, почему алгоритм, который включает погружение на 2 , работает. Давайте выполним шаги, описанные выше, и переместим 2 в левую часть выражений:

        Таким образом, вы можете увидеть, как мы пришли к алгоритму, описанному в начале. Мы также можем поместить вычисления для этих четырех шагов в одно представление, подобное этому 9.0003

        Убедитесь, что вы понимаете, как мы пришли к этому представлению, так как оно понадобится нам при изучении того, как работает алгоритм преобразования из двоичного в десятичный.

        Преобразование десятичной дроби в двоичную

        Чтобы показать, почему мы умножаем на 2 и берем целую часть при преобразовании дробей в двоичную, я также буду использовать base-q форму расширения для дробей. Я буду использовать дробное число 0,375 из первой части статьи. Аналогично целой части, давайте представим, что мы не знаем, как это число представлено в двоичном виде, и запишем его, заменив неизвестные цифры на 9. 0080 x :

        Как и в случае с целыми числами, наша задача состоит в том, чтобы найти все x , выделив x . Давайте посмотрим, как мы можем это сделать. Первое, на что следует обратить внимание, это то, что отрицательные степени числа 2 дают нам дроби со знаменателем числа 2 с положительными степенями. Давайте перепишем приведенное выше выражение:

        Сразу видно, что мы можем просто вынести 1/2 в правой части выражения. Сделаем так:

        и тогда мы можем переместить 1/2 в левую часть

        Хорошо, здесь мы выделили x1 2 1 либо 2 , и мы знаем, что это может быть 90 или 0 . Чтобы определить, какая у него цифра, давайте посмотрим на оставшиеся слагаемые:

        Давайте подумаем, насколько большой может быть сумма этих чисел. Если максимальное количество цифр x равно 1, то мы можем просто заменить x с единицами и запишите сумму в виде:

        Ну, это геометрический ряд дробей, и сумма такого ряда лежит в границах [0 < сумма < 1], поэтому максимальное количество таких сумма может дать нам 1. Давайте теперь снова посмотрим на наше выражение:

        Теперь здесь должно быть ясно, что если правая часть меньше 1, то x1 не может быть равно 1 и, следовательно, равно 0 , а оставшаяся часть равна 0,75 .

        Именно так выглядит первый шаг алгоритма, представленного в начале:

        Отнимем дробную часть от 0,75 и выделим еще 1/2 , чтобы выделить x2

        :

        :

        и переместить 1/2 влево:

        Теперь, если x2 равно 0 , то сумма левой части выражения не может быть больше 1 , но левая часть равна 1,5 2 , поэтому х 2 должно быть 1 а оставшаяся часть 0. 5 . Давайте запишем это:

        Опять же, это следует схеме в алгоритме, представленном в начале:

        Повторим те же действия для оставшейся дробной части 0,5 .

        Используя ту же логику, что и выше, мы можем видеть, что x3 равно 1 и нет оставшейся дробной части:

        вот как выглядит наш последний шаг:

        Итак, давайте еще раз выпишем все шаги:

        Это именно тот алгоритм, который я представил в начале. Точно так же, как мы делали с целыми числами, мы также можем поместить вычисления для этих трех шагов в одно представление, например:

        Опять же, важно, чтобы вы полностью усвоили это представление, так как оно понадобится нам при изучении двоичного преобразования в десятичное.

        Почему не все дроби могут быть конечно представлены в двоичной системе

        Тот факт, что некоторые дроби, представленные конечно в десятичной системе, не могут быть представлены конечно в двоичной системе, является неожиданностью для многих разработчиков. Но именно эта путаница лежит в основе кажущегося странным результата прибавления 0,1 к 0,2. Так что же определяет, может ли дробь быть конечно представлена ​​в системе счисления? Это довольно сложно. Но базовая версия такова — для конечного представления числа знаменатель в дроби должен быть степенью основания системы. Например, для системы с основанием 10 знаменатель должен быть степенью 10, поэтому мы можем конечно представить 0,625 в десятичном виде:

        и не может конечно представлять 1/3:

        То же самое касается системы с основанием 2:

        Но если мы проверим 0,1, знаменатель будет равен 10, и это не степень 2, поэтому 0,1 будет бесконечная дробь в двоичной системе. Давайте посмотрим на это, используя алгоритм, который мы изучили выше:

        Мы можем продолжать делать это бесконечно, но давайте запишем это в виде периодической цепной дроби:

        Преобразование двоичного целого числа в десятичное

        Я собираюсь использовать то же самое двоичное целое число 1011 из первого раздела, чтобы показать вам, почему работает алгоритм умножения на 2. Здесь мы также будем использовать расширение base-q от числа . Давайте запишем это в такой форме:

        Поскольку все слагаемые кратны 2 , мы можем продолжать выносить на множители 2 , пока частное не станет равным нулю. Давайте сделаем так:

        Теперь, если вы просто будете следовать порядку математических операций, вы получите точно такие же шаги, как я показал в начале, а именно:

        Таким образом, 1011 в двоичном виде равно 11 в десятичном виде.

        Преобразование двоичной дроби в десятичную

        Вот мы и подошли к последнему алгоритму. Наверное, вы уже сами разобрались с механикой, стоящей за этим. Если нет, давайте посмотрим, почему это работает. Расширение числа base-q от числа также является ключевым здесь. Число 0,1011 возьмем из первого раздела. Запишем в развернутом виде:

        Опять же, поскольку все слагаемые кратны 1/2 , мы можем продолжать разлагать 1/2 до тех пор, пока не останется дробной части. Сделаем так:

        Следуя порядку математических операций, получается алгоритм, описанный в начале:

        Таким образом, 0,1011 в двоичном виде равно 0,6875 в десятичном.

        Двоичные дроби — Electronics-Lab.com

        Двоичные дроби

        Число, представляющее значение, может состоять из двух частей: целой и дробной части. Разделение между этими двумя частями представлено точкой (точкой). Дробная часть представляет собой значение между двумя целыми числами для выражения точности. Число также может быть представлено только целым числом без дроби, что просто зависит от степени точности, необходимой для представления значения. Число с дробной частью может принадлежать любой системе счисления, т. е. десятичной, двоичной, шестнадцатеричной и т. д.

        Давайте сначала рассмотрим дроби в широко используемой десятичной системе счисления, которая является системой счисления с основанием 10. Будучи числом с основанием 10, он может выбрать любое из десяти (10) возможных чисел для представления значения цифры. Десять (10) возможных чисел: 0, 1, 2, 3, 4, 5, 6, 7, 8 и 9. Кроме того, каждая десятичная цифра имеет позиционный вес, который она несет, который увеличивается на десять (10) раз для каждого позиционного шага справа налево. И наоборот, позиционный вес каждой десятичной цифры уменьшается в десять (10) раз с каждым позиционным шагом слева направо. В выражении это может быть представлено как (10 n ), где «n» представляет позицию десятичной цифры. При движении слева направо позиции за пределами «n = 0» представлены отрицательными значениями, т. е. -1, -2, -3, … и т. д., и поэтому их позиционный вес уменьшается на десять (10) с каждым шаг. Эти отрицательные позиционные веса вместе с соответствующими цифровыми значениями составляют дробное число . Разделение между положительными («n=0») и отрицательными позиционными («n=-1») значениями представлено десятичной точкой .

        Из предыдущих статей известно, что данное число является взвешенной суммой значений его цифр и относительных позиционных весов. Это можно выразить следующим образом:

        Пример дробного числа

        Рассмотрим пример (123,456 10 ) , который представляет собой десятичное число, состоящее из целой и дробной частей, разделенных десятичной точкой. Числа слева и справа от десятичной точки (.) составляют целое число и дробных частей соответственно. В соответствии с этим 123 10 является целой частью, тогда как 456 10 составляет дробную часть данного (123,456 10 ) числа. Позиционный вес каждой цифры зависит от ее относительного положения относительно десятичной точки. Это показано ниже:

        Позиционные веса цифр 1, 2 и 3 слева от десятичной точки:0650 0 (1) соответственно. Точно так же цифры 4, 5 и 6 справа от запятой имеют позиционные веса 10 -1 (0,1), 10 -2 (0,01) и 10 -3 (0,001) соответственно. .

        Состав числа (123,456 10 ) с использованием относительных позиционных весов цифр объясняется следующим образом:

        Концепция позиционной записи для представления относительного веса каждой десятичной цифры может быть применена к другим системам счисления. Однако каждая система нумерации использует другую базу или 9.0012 множитель значение. Например, Octal использует систему счисления по основанию 8, и каждый позиционный шаг увеличивает или уменьшает вес в восемь (8) раз в зависимости от направления шага. Аналогично, в двоичных числах каждая двоичная цифра или бит имеет весовую разницу в два (2) раза по сравнению с соседним битом.

        Двоичные дроби

        Двоичное число использует систему счисления с основанием 2 , и каждая двоичная цифра (бит) имеет два варианта представления своего значения: «0» и «1». Как и десятичная цифра, каждая двоичная цифра (бит) имеет связанный с ней позиционный вес относительно двоичной точки (десятичной точки в случае десятичной цифры). Вес двоичных разрядов увеличивается в два (2) раза на каждый шаг разряда при движении справа налево. Более того, двоичные числа без знака также могут иметь дробные числа, т.е. прямо до двоичной точки. Вес двоичных цифр, представляющих дробные числа, уменьшается в два раза на каждом шаге при движении вправо от двоичной точки. Это означает, что двоичные цифры имеют позиционный вес 9.0012 2 0 , 2 1 , 2 2 , и т. д. , начиная с левого двоичного числа, т. е. номера позиций 0, 1, 2 и т. д. соответственно. Точно так же двоичные цифры имеют позиционный вес 2 -1 , 2 -2 , 2 -3 и т. д. , начиная непосредственно справа от двоичной точки.

        Относительные позиционные веса двоичных цифр поясняются ниже:

        Учитывая двоичную дробь 0,1101 2 , считается, что позиционный вес каждой двоичной цифры или бита, составляющих двоичную дробь, составляет ее десятичный эквивалент. Это показано ниже:

        Двоичная дробь 0,1101 2 дает эквивалентную десятичную дробь 0,8125 10 .

        Примеры двоичных дробей

        Следующие двоичные дроби преобразуются в эквивалентные им десятичные дроби с помощью описанного выше метода.

        Сверху видно, что биты двоичной дроби имеют вес, уменьшающийся в два (2) раза с каждым позиционным шагом вправо или, наоборот, вес становится вдвое (1/2 раза) в пересчете на десятичный эквивалент.

        Преобразование десятичной дроби в двоичную

        Для преобразования десятичной дроби в ее двоичный эквивалент применяется метод, использованный ранее для преобразования десятичной дроби в двоичную. Однако вместо многократного деления на два (2) применяется умножение дробной части.

        Десятичное число, состоящее из целой и дробной части, требует преобразования каждой части отдельно. Для целочисленной части требуется метод повторного деления на 2, тогда как для дробной части требуется повторное умножение на 2.

        Обсуждавшийся ранее метод преобразования десятичной дроби в двоичную применяется только к целой части, составляющей положительные веса. Целая часть десятичного числа многократно делится на два (2), отмечая остаток в обратном порядке (от LSB к MSB), пока разделенное десятичное значение не станет «0». Это показано ниже путем преобразования 238 10 в его двоичный эквивалент.

        Следовательно, двоичный эквивалент числа 238 10 , состоящий только из целой части, равен 11101110 2 .

        Аналогично, двоичный эквивалент дробной части десятичного числа находится повторным умножением на 2 и отметкой переносов вниз в порядке пересылки, пока дробная часть не станет равной «0». Данную десятичную дробь, которая будет заведомо меньше 1, умножают на два (2), в результате чего получается значение, которое будет либо больше, либо меньше единицы (1). Если в результате умножения получается значение меньше единицы (1), перенос вперед не выполняется, т. е. «0», и процесс умножения повторяется с отметкой «0», пока результат не станет равным или больше единицы. Умножение, в результате которого получается значение, большее или равное единице (1), производит перенос вперед, т. е. «1», и процесс умножения повторяется только для его дробной части (для случаев, когда умножение приводит только к значению больше единицы). В случае, если значение становится равным единице (1), дальнейшее умножение не применяется.

        В некоторых случаях может показаться, что повторное умножение не сходится к решению, т. е. дает точную единицу или результирующую дробь, равную «0». В таких случаях дробные числа имеют бесконечную длину, а умножения или числовые биты ограничены для получения желаемой степени точности.

        Примеры преобразования десятичной дроби в двоичную

        Следующие примеры преобразования десятичной дроби в двоичную объясняют вышеупомянутый метод.

        Двоичный эквивалент 0,625 10 равно 0,101 2 . Точно так же смешанное десятичное число (43,375 10 ) можно преобразовать в его двоичный эквивалент, как показано ниже:

        Таким образом, двоичный эквивалент смешанного десятичного числа (43,375 10 2) 101011.011 2.

        Заключение

        • В двоичных числах используется система счисления с основанием 2, поэтому вес каждой двоичной цифры (бита) равен 2 n , где «n» представляет позицию этого бита относительно двоичной точки.
        • Биты слева от двоичной точки (.) имеют позиции в положительной последовательности, т. е. 0, 1, 2, 3 и т. д. Тогда как справа имеют позиции в отрицательной последовательности, т. е. -1, -2, -3, -4 и т. д., которые составляют двоичную дробь.
        • Для позиций прямой последовательности вес каждого бита увеличивается в два раза на каждый шаг увеличения позиции. В то время как для позиций обратной последовательности (двоичная дробь) вес каждого бита уменьшается в два раза для каждого шага уменьшения позиции.

      Буквенные алгебраические выражения: Ошибка 403 — доступ запрещён

      Алгебраические выражения, коэффициент, виды выражений

      • Коэффициент
      • Виды выражений

      Алгебраическое выражение — это запись, составленная со смыслом, в которой числа могут быть обозначены и буквами, и цифрами. Также она может содержать знаки арифметических действий и скобки.

      Любую букву, обозначающую число, и любое число, изображённое с помощью цифр, принято считать в алгебре также алгебраическим выражением.

      Алгебраические выражения, входящие в состав формул, могут применяться к решению частных арифметических задач, если в них заменить буквы данными числами и произвести указанные действия. Число, которое получится, если взять вместо букв какие-либо числа и произвести над ними указанные действия, называется численной величиной алгебраического выражения. Из этого легко сделать вывод, что одно и то же алгебраическое выражение при различных значениях входящих в него букв может иметь различные числовые величины.

      Примеры:

      1) Выражение

      am + bn,

      при  a = 2,  m = 5,  b = 1,  n = 4  вычисляется:

      2 · 5 + 1 · 4 = 14,

      а при  a = 3,  m = 4,  b = 5,  n = 1  вычисляется:

      3 · 4 + 5 · 1 = 17  и т. д.

      2) Выражение

      abс,

      при  a = 1,  b = 2,  c = 3  равно:

      1 · 2 · 3 = 6,

      а при  a = 2,  b = 3,  c = 4  равно:

      2 · 3 · 4 = 24  и т. д.

      Коэффициент

      Коэффициент — это числовой множитель алгебраического выражения, представляющего собой произведение нескольких сомножителей. Коэффициент в выражении ставится перед всеми остальными буквенными множителями. Таким образом,

      произведение чисел  abcd,  4  записывается так: 4abcd;

      произведение чисел  mn,  ,  p  записывается так:   .

      Числа  4  и    — это коэффициенты. Очевидно, что

      4abcd = abcd + abcd + abcd + abcd

      и точно также

      .

      Итак, коэффициент показывает, сколько раз целое алгебраическое выражение или известная его часть берется слагаемым.

      Если в алгебраическом выражении нет числового множителя, то подразумевается, что коэффициент равен единице, так как

      a = 1 · a;     bc = 1 · bc

      и так далее.

      Виды выражений

      Алгебраическое выражение, в которое не входят буквенные делители, называется целым, в противном случае дробным или алгебраической дробью.

      Пример.

      Целые алгебраические выражения:

      7a2b,     a22bc .
      3

      Дробные алгебраические выражения:

      a2 ,    mn
      b3m + n .

      Выражения, не содержащие корней, называются рациональными, а содержащие корни — иррациональными или радикальными. Например, все выражения, приведённые выше, являющиеся целыми или дробными, так же можно назвать и рациональными.

      a ,   53c + amn   — иррациональные или радикальные выражения.

      023. Алгебраические выражения

      Алгебраическое выражение – это выражение, которое состоит из чисел, переменных и математических знаков. Выражение может содержать скобки, рациональную степень переменной (с целым или дробным показателем), знак модуля.

      Например, алгебраические выражения – это: .

      Область допустимых значений (ОДЗ) алгебраического выражения – это такие значения переменных, при которых это выражение имеет смысл.

      Пример 3. Найдите ОДЗ алгебраического выражения: .

      Решение. Если , то выражение не имеет смысла.

      Ответ. ОДЗ: .

      Пример 4. Найдите ОДЗ выражения: .

      Решение. Из условия имеем:. Это значит, что если или , то выражение не имеет смысла.

      Ответ. ОДЗ: .

      Рассмотрим рисунок 3.1 подробнее, для этого дадим характеристику каждому компоненту рисунка.

      Алгебраические выражения могут быть рациональными и иррациональными.

      Рациональное выражение – это выражение, в котором содержатся действия сложения, вычитания, умножения, деления, возведения в степень (здесь показатель степени – это натуральное число).

      Например, ; ; – это рациональные выражения.

      Рациональные выражения могут быть Целыми и Дробными.

      Целое рациональное выражение не содержит деления на буквенное выражение. Например, ; – это целые рациональные выражения.

      Целые рациональные выражения подразделяются на одночлены и многочлены.

      Одночлен – это произведение числового коэффициента на натуральную степень переменных.

      Например, ; ; – это одночлены, где ; ; – это числовые коэффициенты; ; ; – это буквенные выражения.

      Степень одночлена – это сумма показателей степеней всех переменных одночлена.

      Например, одночлен – это одночлен шестой степени одночлен – это одночлен четвертой степени 7 – это одночлен нулевой степени.

      Одночлен имеет Стандартный вид, если числовой коэффициент стоит на первом месте (перед буквенным выражением), а неизвестные множители записаны в алфавитном порядке.

      Одночлены называются Подобными, если они имеют одинаковые буквенные выражения.

      Привести подобные одночлены (члены) – это значит найти их сумму или разность.

      Например, ; ; – это подобные одночлены.

      Пример 5. Приведите подобные члены: .

      Решение. ; ; .

      Ответ. .

      Многочлен – это алгебраическая сумма одночленов (их сумма или разность).

      Например, – это многочлен.

      Степень многочлена – это степень его старшего члена.

      Например, многочлен – это многочлен четвертой степени; многочлен – это многочлен пятой степени.

      Дробное рациональное выражение содержит деление на выражение с переменными. Дробное рациональное выражение называют алгебраической дробью.

      Например, ; – это алгебраические дроби.

      Используют и другое определение алгебраической дроби.

      Выражение вида , где и – это многочлены или одночлены, называется Алгебраической дробью.

      Область допустимых значений (ОДЗ) Алгебраической дроби это множество значений переменной, при которых ее знаменатель не равен нулю ().

      Алгебраическая дробь равна нулю, если ее числитель равен нулю, т. е. .

      Если в алгебраическом выражении используется возведение переменных в дробную степень (извлечение корня из переменных), то такое алгебраическое выражение называется Иррациональным.

      Например, ; – это иррациональные выражения.

      Алгебраические выражения могут быть рациональными и иррациональными.

      Рациональные выражения разделяются на целые и дробные.

      Целые рациональные выражения состоят из одночленов и многочленов.

      Дробные рациональные выражения включают в себя алгебраические дроби.

      < Предыдущая   Следующая >

      буквенных уравнений | ChiliMath

      Буквенные уравнения , проще говоря, представляют собой уравнения, содержащие две или более переменных. Ваша цель состоит в том, чтобы решить только одну переменную по отношению к другим. Если вы знаете, как решать регулярные уравнения, то я гарантирую вам, что решение буквальных уравнений будет легкой задачей.

      Что такое буквальное уравнение?

      Буквенное уравнение — это уравнение, включающее более одной переменной. Более того, переменная или «литерал» — это математический символ, представляющий произвольное значение или число. Буквы в алфавите обычно используются для обозначения таких переменных, как a, b, c, x, y и z. Решить буквальное уравнение означает выразить одну переменную по отношению к другим переменным в уравнении.


      Ключевая стратегия решения буквальных уравнений

      «Суть» решения буквального уравнения в том, чтобы изолировать или оставить отдельно определенную переменную с одной стороны уравнения (слева или справа), а остальные – с противоположной стороны.

      Если вы знаете, как решать обычные одношаговые уравнения, двухшаговые уравнения и многошаговые уравнения, процесс решения буквенных уравнений очень похож.

      Поэтому вас не должны пугать буквальные уравнения, потому что у вас уже могут быть навыки для их решения. Это просто вопрос практики и знакомства.

      Итак, ключевая идея выглядит так. Обратите внимание, что переменная, которую вы хотите решить, изолирована на одной стороне уравнения. В данном случае она находится на левой стороне. Обратите внимание, что переменная \color{red}\Large{x} сама по себе находится на одной стороне уравнения, а остальные — на противоположной.


      Давайте рассмотрим несколько примеров!

      Пример одношагового буквального уравнения

      Пример 1 : Найдите s в буквальном уравнении P = 4s.

      Помните эту формулу? Это периметр квадрата, где P означает периметр, а s – размер одной стороны квадрата. Таким образом, чтобы получить периметр квадрата, мы имеем P = s + s + s + s = 4s.

      Чтобы найти s, нам нужно избавиться от коэффициента 4, который умножает s. Обратное умножение — это деление, поэтому мы должны делить обе части на 4!

      • Мы можем изолировать переменную s с правой стороны.
      • Разделите обе части на 4.
      • Упрощение. Теперь, когда переменная \color{red}\Large{s} одна в правой части уравнения, мы закончили!

      Примеры двухшаговых литеральных уравнений

      Пример 2 : Найдите L в буквальном уравнении P = 2L + 2W.

      Буквенное уравнение, упомянутое выше, также является формулой для определения периметра прямоугольника, где: P = периметр, L = длина и W = ширина. Можно изолировать переменную L в правой части. Однако почему бы не перевернуть уравнение, чтобы мы могли оставить только переменную L слева? Что ж, звучит как план!

      Пусть вас не пугает внешний вид. Просто сосредоточьтесь на вещах, которые вы хотите сделать, то есть на решении для L, а остальные шаги последуют за вами.

      • Мы хотим решить L, верно?
      • Переверните уравнение , чтобы изолировать переменную в левой части.
      • Вычтите обе стороны на 2W.
      • Упрощение.
      • Разделите обе части на 2.
      • Упрощение, теперь L стоит отдельно – решено!

      Пример 3 : Найдите x в приведенном ниже буквальном уравнении.

      Что делает это буквальное уравнение интересным, так это то, что мы собираемся изолировать переменную, которая является частью числителя дроби. Я не уверен, помните ли вы, что всякий раз, когда вы видите что-то подобное, постарайтесь сначала избавиться от знаменателя. Это значительно упрощает весь процесс решения.

      Поскольку знаменатель 3 делит выражение «x − y», противоположная операция, которая может его отменить, — умножение. Имеет смысл сначала умножить обе части на 3, а затем добавить на «y», чтобы оставить x отдельно. Не так уж плохо, верно?

      • Хорошо, мы хотим найти x. Изолируем его с правой стороны.
      • Начните с умножения обеих сторон на 3, что является знаменателем дроби.
      • Упрощение. Это хорошо, знаменатель исчез.
      • Добавить обе стороны по у. Это единственный способ убрать − y с правой стороны.
      • Упрощение, x теперь счастлив сам по себе. Сделанный!

      Примеры многошаговых буквенных уравнений

      Пример 4 : Решите для C в буквальном уравнении ниже.

      Это формула, используемая для преобразования меры температуры в единицах Цельсия в шкалу Фаренгейта. Обратите внимание: чтобы найти значение F (по Фаренгейту), нам нужно подставить некоторое значение C (по Цельсию).

      Однако можем ли мы также использовать данную формулу для нахождения градусов Цельсия всякий раз, когда задано значение Фаренгейта?

      Абсолютно да!

      В конце концов, это буквальное уравнение, поэтому можно выразить C через F. Это то, что мы собираемся сделать сейчас…

      • Все глаза прикованы к C. Цель состоит в том, чтобы изолировать его.
      • Избавимся от 32 справа, вычтя обе части на 32.
      • Это выглядит чище после упрощения.
      • Затем умножьте обе части на 5, чтобы сократить знаменатель 5 под 9C.
      • Мы на месте! Я предлагаю , а не , чтобы распределить 5 на (F — 32).
      • Еще один шаг, разделите обе части на 9, чтобы окончательно изолировать переменную справа.
      • Вот и все! Мы решили для C.

      Пример 5 : Найдите h в буквальном уравнении 3h + g = 5h − hg.

      Это действительно интересно! Некоторые из вас могут подумать, что невозможно изолировать переменную h, поскольку она находится почти в трех местах: одно h слева и два справа. Ну не сдавайся еще! Открою вам небольшой «секрет».

      Используйте метод факторинга, чтобы выбрать эту переменную h   из группы. Но прежде чем вы сможете разложить h, убедитесь, что вы переместили все h на одну сторону уравнения.

      Поскольку у нас есть два члена h в правой части, мы могли бы также переместить член 3h слева на другую сторону.

      • Мы хотим h изолировать, верно?
      • Сохраняем все наши h-члены справа. Мы можем сделать это, вычитая обе стороны на 3h.
      • После упрощения замечательно видеть, что все наши h-термины находятся только справа.
      • Очевидно, что шаг должен включать в себя факторизацию h.

      Вау, это здорово! Всего одиночных ч с правой стороны.

      • Само по себе выделение h означает, что мы должны избавиться от выражения (2−g).

      Разделите обе части на (2 − g).

      • Сделайте несколько отмен с правой стороны.
      • Вот оно! Мы решили для h.

      Пример 6 : Найдите x в приведенном ниже буквальном уравнении.

      Самый простой способ решить это буквальное уравнение — выполнить перекрестное умножение . При этом знаменатели в обеих частях уравнения должны исчезнуть.

      С этого момента мы можем применить ту же стратегию из примера 5, чтобы найти x , которая включает в себя сбор всех членов x на одной стороне уравнения, а затем, как мы надеемся, факторизацию x.

      • В этом уравнении у нас есть два x с обеих сторон уравнения. Что еще более важно, они расположены в позиции числителя.
      • Мы хотим, чтобы знаменатели исчезли, поэтому без колебаний мы должны применить метод перекрестного умножения. Затем просто примените распределительное свойство к обеим частям уравнения.
      • На этом этапе мы решаем, где хранить или собирать все наши крестики. Для этого примера давайте оставим их слева.

      Начните с , избавьтесь от -5x справа, добавив 5x с обеих сторон.

      • Вот так это выглядит после упрощения. Следующим шагом будет работа с 3xy с правой стороны. Мы также хотим переместить его влево.
      • Вычтите обе стороны на 3xy. Это должно оставить все наши крестики слева.
      • Не забудьте написать 0 справа!
      • Теперь -6 слева нужно переместить на правую сторону. Мы можем сделать это, добавив 6 к обеим сторонам.
      • Становится лучше! У нас есть все наши x термины слева. Похоже, что мы можем исключить x.
      • Ага! Мы только что сделали!
      • Наконец, чтобы решить x, мы должны разделить обе части на выражение (8 − 3y). Выполните некоторые отмены.
      • Готово!

      Пройди тест !

      Викторина по буквальным уравнениям


      Вам может быть интересно:

      Решение одношаговых уравнений

      Решение двухэтапных уравнений

      Решение многошаговых уравнений

      Решение буквенных уравнений: обзор и примеры

      Смотри, ты идешь! Теперь вы научились решать одношаговые уравнения, двухшаговые уравнения и многошаговые уравнения.

      Теперь пришло время обсудить решение буквальных уравнений .

      На этом уроке мы дадим определение буквенным уравнениям, рассмотрим примеры буквенных уравнений, а также научимся решать буквальные уравнения. Давайте буквально взволнован, чтобы начать!

      Что мы рассматриваем

      Что такое буквальное уравнение?

      Напомним из нашего предыдущего исследования, что уравнение — это математическое предложение, в котором используется знак равенства = , чтобы показать, что два выражения равны. В отличие от других уравнений, с которыми вы уже работали, буквальные уравнения состоят в основном из букв и переменных.

      Многие из буквальных уравнений, с которыми вы работали в своей жизни, были формулы. Хотя эти уравнения будут выглядеть иначе, чем наши обычные уравнения, они по-прежнему подчиняются тем же правилам решения.

      Вернуться к оглавлению

      Примеры буквенных уравнений

      Хотя идея уравнений, состоящих в основном из букв, может показаться чуждой, вы много раз использовали буквальные уравнения в своей жизни. Вот несколько примеров буквальных уравнений, с которыми вы уже работали в своей жизни:

      Площадь прямоугольника

      A = b \cdot h

      Длина окружности

      C = \pi \cdot D

      Формула простых процентов

      I = p \cdot r \cdot t

      Каждая буква (или переменная) в буквальном уравнении имеет особое значение и изменяется от задачи к задаче.

      Вернуться к оглавлению

      Как решать буквенные уравнения

      Решение буквенных уравнений следует тем же правилам, что и решение одношагового или двухэтапного уравнения. Идея «решения» буквального уравнения, по сути, означает, что мы переставляем буквы (или переменные), чтобы изолировать новую переменную. Буквальное уравнение «решено», когда интересующая переменная находится одна на одной стороне уравнения.

      Ознакомьтесь с лицензиями школы Альберта !

      Подобно решению уравнений, мы будем использовать обратные операции, чтобы изолировать переменную саму по себе. Вот примеры обратных операций:

      \text{Сложение} \leftrightarrow \text{Вычитание}

      \text{Умножение} \leftrightarrow \text{Деление}

      Вот несколько примеров решения буквенных уравнений:

      Пример 1

      Найдите h в следующем буквальном уравнении:

        А = b \cdot h

      Помните эту формулу? Как сказано выше, это площадь прямоугольника. Как отмечалось ранее, в буквенных уравнениях в основном используются буквы и переменные. Если бы это было простое уравнение, такое как 10 = 2x, мы бы просто разделили обе части на 2, чтобы получить мой окончательный ответ.

      При «решении» буквенных уравнений мы следуем тем же правилам, что и простые уравнения. Следовательно, чтобы найти h в этом уравнении, нам нужно выделить его отдельно. Поэтому мы разделим обе части на b .

      \dfrac{A}{b} = \dfrac{b \cdot h}{b}

      Это изолирует h , что даст нам ответ: 

      h = \dfrac{A}{b}

      Пример 2: 

      Хотя формулы являются распространенным примером буквенных уравнений, не все буквальные уравнения являются формулами. Мы также можем изменить и «решить» буквальное уравнение для любой переменной. Например: 

      Найдите m в следующем уравнении: 

      x = m + n Исходное уравнение
      x \textcolor{red}{- n} = m + n \textcolor{red}{- n} Вычесть n с обеих сторон
      x — n = m m теперь изолировано
      m = x — n

      Несмотря на то, что в уравнении не было чисел, мы «решили» буквальное уравнение для m .

      Вернуться к оглавлению

      Многошаговые буквенные уравнения

      Пример 1

      Не все буквальные уравнения решаются только за один шаг. Вот пример использования нескольких шагов для решения буквенных уравнений. 92}

      Теперь у нас есть r, изолированный сам по себе, что дает нам новое буквальное уравнение:

      r = \sqrt{\dfrac{V}{\pi h}}

      Пример 2

      Вот пример буквального уравнения, которое не является формулой, но которое мы можем решить для переменной.

      Найдите x в следующем уравнении:

      4(x + y) = P

      Есть два способа решить эту проблему. Первый метод состоит в том, чтобы рассматривать его как уравнение и распределять 4 , а затем решать:

      4x + 4y = Р

      Затем мы можем вычесть 4y с каждой стороны:

      4x + 4y \textcolor{red}{- 4y} = P \textcolor{red}{ — 4y}

      Затем нам нужно разделить каждую сторону на 4 :

      \dfrac{4x}{4} = \dfrac{P — 4y}{4}

      Наконец, нам нужно упростить наше уравнение:

      x = \dfrac{P}{4} — \dfrac{4y}{4}

      х = \dfrac{P}{4} — у

      Теперь мы, наконец, нашли x в буквальном уравнении. Давайте посмотрим, как можно решить уравнение, не упрощая в конце.

      В другом методе мы можем просто разделить на 4 в начале, чтобы избежать использования свойства распределения. Например: 

      \dfrac{4(x + y)}{4} = \dfrac{P}{4}

      Тогда нам просто нужно вычесть y с каждой стороны:

      x + y \textcolor{red}{- y} = \dfrac{P}{4} \textcolor{red}{ — y}

      Таким образом, мы получаем:

      x = \dfrac{P}{4}- y

      Обратите внимание, что уравнение уже упрощено, и никаких других шагов не требуется.

      Вернуться к оглавлению

      Вот короткое видео, демонстрирующее, как решать буквенные уравнения:

      Буквенные уравнения с дробями

      Давайте поработаем над некоторыми примерами буквенных уравнений с дробями!

      Ознакомьтесь с лицензиями школы Альберта !

      Пример 1

      Многие буквенные уравнения и формулы в той или иной степени содержат дроби. Например, вот формула объема сферы:

      93}
      Кубический корень с обеих сторон
      r = \sqrt[3]{\dfrac{3V}{4\pi}}

      Теперь, когда r изолировано, мы успешно нашли r .

      Пример 2

      Что произойдет, если мы просто захотим изменить уравнение для другой переменной?

      Например, решите следующее уравнение для x :

      y = \dfrac{x}{4} — \dfrac{1}{8}

      Обратите внимание, что в уравнении две переменные, и наша конечная цель по-прежнему состоит в том, чтобы изолировать x . Помните, мы можем убрать все дроби за один ход, умножив все члены на 9.0003 Наименьший общий знаменатель .

      В этом буквальном уравнении наименьший общий знаменатель равен 8 . Следовательно, мы умножим каждое слагаемое на 8.

      8 \cdot y = 8 \cdot \dfrac{x}{4} — 8 \cdot \dfrac{1}{8}

      Это даст нам уравнение, в котором больше нет дробей:

      8y = 2x — 1

      Затем продолжайте решать, как обычное уравнение:

      8y = 2x — 1 Исходное уравнение
      8y \textcolor{red}{+ 1} = 2x — 1 \textcolor{red}{+ 1} Добавить по 1 с каждой стороны
      8y + 1 = 2x Упростить
      5 \fracd {8y + 1}{\textcolor{red}{2}} = \dfrac{2x}{\textcolor{red}{2}} Разделите каждую сторону на 2
      \dfrac{8y}{2} + \dfrac{1}{2} = x Упрощение
      4y + \dfrac{1}{2} = x Упрощение

      Теперь, когда мы изолировали x отдельно, мы правильно «решил» буквальное уравнение.

    © 2015 - 2019 Муниципальное казённое общеобразовательное учреждение «Таловская средняя школа»

    Карта сайта